You are on page 1of 45

Assignment:

1. General principles and Books 1 and 2


2. And the following cases:

EMPLOYER-EMPLOYEE RELATIONSHIP

A. Tests of employer-employee relationship (p. 138 GN)

1. Traditional Four-Fold Test


The four–fold test (indicia of determination):
1. Selection and engagement of the employee;
2. Payment of wages;
3. Power of dismissal; and
4. Power of control. (Azucena, Vol. 1, 2016 p. 189)
It is the so-called “control test” that is the most important element.

The control test calls merely for the existence of the right to control the manner of doing the work,
not the actual exercise of the right.

The main determinant therefore is whether the rules set by the employer are meant to control not
just the results but also the means and methods.

NOTE: However, in certain cases the control test is not sufficient to give a complete picture of the
relationship between the parties, owing to the complexity of such a relationship where several
positions have been held by the worker. The better approach is to adopt the two-tiered test.

2. Two-Tiered Test
1. the putative employer’s power to control the employee with respect to the means and
methods by which the work is to be accomplished (Four-fold test); and
2.. the underlying economic realities of the activity or relationship (economic reality test).

3. Economic Dependency Test


Under the economic realities test, the standard of economic dependence is whether the person is
dependent on the prospective employer for his continued employment in that line of
business.

The determination of the relationship between Er and Ee depends upon the circumstances of
the whole economic activity, such as:
1. The extent to which the services performed are an integral part of the Er’s business;
2. The extent of the worker’s investment in equipment and facilities;
3. The nature and degree of control exercised by the Er;
4. The worker’s opportunity for profit and loss;
5. The amount of initiative, skill, judgment, or foresight required for the success of the
claimed independent enterprise;
6. The permanency and duration of the relationship between the worker and Er; and
7. The degree of dependency of the worker upon the Er for his continued employment in
that line of business. (Francisco v. NLRC, G.R. No. 170087, Aug. 31, 2006)
B. Analysis of the jurisprudential doctrines:

1. Existence of employer-employee relationship cannot be repudiated in the


employment contract

Century Properties, Inc. v. Babiano, G.R. No. 220978, July 05, 2016
Labor Law; Employer-Employee Relationship; Control Test; The control test is commonly regarded
as the most important indicator of the presence or absence of an employer-employee relationship.—Anent
the nature of Concepcion’s engagement, based on case law, the presence of the following elements evince
the existence of an employer-employee relationship: (a) the power to hire, i.e., the selection and
engagement of the employee; (b) the payment of wages; (c) the power of dismissal; and (d) the
employer’s power to control the employee’s conduct, or the so called “control test.” The control test is
commonly regarded as the most important indicator of the presence or absence of an employer-employee
relationship. Under this test, an employer-employee relationship exists where the person for whom the
services are performed reserves the right to control not only the end achieved, but also the manner and
means to be used in reaching that end.

Same; Same; The existence of employer-employee relations could not be negated by the mere
expedient of repudiating it in a contract.—While the employment agreement of Concepcion was
denominated as a “Contract of Agency for Project Director,” it should be stressed that the existence of
employer-employee relations could not be negated by the mere expedient of repudiating it in a contract. In
the case of Insular Life Assurance Co., Ltd. v. NLRC (4th Division), 287 SCRA 476 (1998), it was ruled
that one’s employment status is defined and prescribed by law, and not by what the parties say it should
be, viz.: It is axiomatic that the existence of an employer-employee relationship cannot be negated by
expressly repudiating it in the management contract and providing therein that the “employee” is an
independent contractor when the terms of the agreement clearly show otherwise. For, the employment
status of a person is defined and prescribed by law and not by what the parties say it should be. In
determining the status of the management contract, the “four-fold test” on employment earlier mentioned
has to be applied. (Emphasis and underscoring supplied) Therefore, the CA correctly ruled that since there
exists an employer-employee relationship between Concepcion and CPI, the labor tribunals correctly
assumed jurisdiction over her money claims.

Facts:

Babiano was hired by CPI as Director of Sales, and was eventually[6] appointed as Vice
President for Sales

His employment contract[7] also contained a "Confidentiality of Documents and


Non-Compete Clause"[8] which, among others, barred him from disclosing confidential
information, and from working in any business enterprise that is in direct competition with
CPI "while [he is] employed and for a period of one year from date of resignation or
termination from [CPI]."

During the same period, Concepcion was initially hired as Sales Agent by CPI and was
eventually[10] promoted as Project Director

As such, she signed an employment agreement, denominated as "Contract of Agency for


Project Director"

Notably, it was stipulated in both contracts that no employer-employee relationship exists


between Concepcion and CPI

After receiving reports that Babiano provided a competitor with information regarding CPFs
marketing strategies, spread false information regarding CPI and its projects, recruited
CPI's personnel to join the competitor, and for being absent without official leave (AWOL)
for five (5) days, CPI, through its Executive Vice President for Marketing and Development,
Jose Marco R. Antonio (Antonio), sent Babiano a Notice to Explain

On February 25, 2009, Babiano tendered[18] his resignation and revealed that he had been
accepted as Vice President of First Global BYO Development Corporation (First Global), a
competitor of CPI

Babiano was served a Notice of Termination[20] for: (a) incurring AWOL; (b) violating the
"Confidentiality of Documents and Non-Compete Clause" when he joined a competitor
enterprise while still working for CPI and provided such competitor enterprise information
regarding CPFs marketing strategies; and (c) recruiting CPI personnel to join a
competitor.[21]

On the other hand, Concepcion resigned as CPFs Project Director through a letter...
respondents filed a complaint[23] for non-payment of commissions and damages against
CPI and Antonio before the NLRC,... For its part, CPI maintained[25] that Babiano is merely
its agent tasked with selling its projects. Nonetheless, he was afforded due process in the
termination of his employment which was based on just causes.[26] It also claimed to have
validly withheld Babiano's commissions, considering that they were deemed forfeited for
violating the "Confidentiality of Documents and Non-Compete Clause."... the Labor Arbiter
(LA) ruled in CPI's favor and, accordingly, dismissed the complaint for lack of merit.[30]... he
NLRC reversed and set aside the LA ruling, and entered a new one ordering CPI to pay
Babiano and Concepcion

While the NLRC initially concurred with the LA that Babiano's acts constituted just cause
which would warrant the termination of his employment from CPI, it, however, ruled that the
forfeiture of all earned commissions of Babiano under the "Confidentiality of Documents and
Non-Compete Clause" is confiscatory and unreasonable and hence, contrary to law and
public policy.

In this light, the NLRC held that CPI could not invoke such clause to avoid the payment of
Babiano's commissions since he had already earned those monetary benefits and, thus,
should have been released to him.

Meanwhile, contrary to the LA's finding, the NLRC ruled that Concepcion was CPI's
employee, considering that CPI: (a) repeatedly hired and promoted her since 2002; (b) paid
her wages despite referring to it as "subsidy"; and (c) exercised the power of dismissal and
control over he... the CA affirmed the NLRC ruling with modification increasing the award of
unpaid commissions to Babiano and Concepcion

The CA held that Babiano properly instituted his claim for unpaid commissions before the
labor tribunals as it is a money claim arising from an employer-employee relationship with
CPI. In this relation, the CA opined that CPI cannot withhold such unpaid commissions on
the ground of Babiano's alleged breach of the "Confidentiality of Documents and
Non-Compete Clause"

The petition is partly meritorious.

Issues:

The core issue for the Court's resolution is whether or not the CA erred in denying CPI's
petition for certiorari, thereby holding it liable for the unpaid commissions of respondents.

Ruling:

Thus, in the interpretation of contracts, the Court must first determine whether a provision or
stipulation therein is ambiguous. Absent any ambiguity, the provision on its face will be read
as it is written and treated as the binding law of the parties to the contract.[54]In the case at
bar, CPI primarily invoked the "Confidentiality of Documents and Non-Compete Clause"
found in Babiano's employment contract[55] to justify the forfeiture of his commissions,
viz.:Confidentiality of Documents and Non-Compete Clause

Finally, if undersigned breaches any terms of this contract, forms of compensation including
commissions and incentives will be forfeited.[56] (Emphases and underscoring supplied)

Verily, the foregoing clause is not only clear and unambiguous in stating that Babiano is
barred to "work for whatsoever capacity x x x with any person whose business is in direct
competition with [CPI] while [he is] employed and for a period of one year from date of [his]
resignation or termination from the company," it also expressly provided in no uncertain
terms that should Babiano "[breach] any term of [the employment contract], forms of
compensation including commissions and incentives will be forfeited."

Indubitably, obligations arising from contracts, including employment contracts, have the
force of law between the contracting parties and should be complied with in good faith.[59]
Corollary thereto, parties are bound by the stipulations, clauses, terms, and conditions they
have agreed to, provided that these stipulations, clauses, terms, and conditions are not
contrary to law, morals, public order or public policy,[60] as in this case.

Therefore, the CA erred in limiting the "Confidentiality of Documents and Non-Compete


Clause" only to acts done after the cessation of the employer-employee relationship or to
the "post-employment" relations of the parties. As clearly stipulated, the parties wanted to
apply said clause during the pendency of Babiano's employment
From the foregoing, it is evidently clear that when he sought and eventually accepted the
said position with First Global, he was still employed by CPI as he has not formally resigned
at that time. Irrefragably, this is a glaring violation of the "Confidentiality of Documents and
Non-Compete Clause" in his employment contract with CPI, thus, justifying the forfeiture of
his unpaid commissions.

Guided by these parameters, the Court finds that Concepcion was an employee of CPI
considering that: (a) CPI continuously hired and promoted Concepcion from October 2002
until her resignation on February 23, 2009,[64] thus, showing that CPI exercised the power
of selection and engagement over her person and that she performed functions that were
necessary and desirable to the business of CPI; (b) the monthly "subsidy" and cash
incentives that Concepcion was receiving from CPI are actually remuneration in the concept
of wages as it was regularly given to her on a monthly basis without any qualification, save
for the "complete submission of documents on what is a sale policy";[65] (c) CPI had the
power to discipline or even dismiss Concepcion as her engagement contract with CPI
expressly conferred upon the latter "the right to discontinue [her] service anytime during the
period of engagement should [she] fail to meet the performance standards,"[66] among
others, and that CPI actually exercised such power to dismiss when it accepted and
approved Concepcion's resignation letter; and most importantly, (d) as aptly pointed out by
the CA, CPI possessed the power of control over Concepcion because in the performance
of her duties as Project Director - particularly in the conduct of recruitment activities, training
sessions, and skills development of Sales Directors - she did not exercise independent
discretion thereon, but was still subject to the direct supervision of CPI, acting through
Babiano.[67]

Therefore, the CA correctly ruled that since there exists an employer-employee relationship
between Concepcion and CPI, the labor tribunals correctly assumed jurisdiction over her
money claims.

Finally, CPI contends that Concepcion's failure to assail the NLRC ruling awarding her the
amount of P470,754.62 representing unpaid commissions rendered the same final and
binding upon her. As such, the CA erred in increasing her monetary award to
P591,953.05.[70]

As a general rule, a party who has not appealed cannot obtain any affirmative relief other
than the one granted in the appealed decision. However, jurisprudence admits an exception
to the said rule, such as when strict adherence thereto shall result in the impairment of the
substantive rights of the parties concerned. In Global Resource for Outsourced Workers,
Inc. v. Velasco:[71]Indeed, a party who has failed to appeal from a judgment is deemed to
have acquiesced to it and can no longer obtain from the appellate court any affirmative relief
other than what was already granted under said judgment. However, when strict adherence
to such technical rule will impair a substantive right, such as that of an illegally dismissed
employee to monetary compensation as provided by law, then equity dictates that the Court
set aside the rule to pave the way for a full and just adjudication of the case.[72

In the present case, the CA aptly pointed out that the NLRC failed to account for all the
unpaid commissions due to Concepcion for the period of August 9, 2008 to August 8,
2011.[73] Indeed, Concepcion's right to her earned commissions is a substantive right which
cannot be impaired by an erroneous computation of what she really is entitled to

In sum, the Court thus holds that the commissions of Babiano were properly forfeited for
violating the "Confidentiality of Documents and Non-Compete Clause." On.the other hand,
CPI remains liable for the unpaid commissions of Concepcion in the sum of P591,953.05.

Principles:

Insular Life Assurance Co., Ltd. v. NLRC,[68] it was ruled that one's employment status is
defined and prescribed by law, and not by what the parties say it should be, viz.:It is
axiomatic that the existence of an employer-employee relationship cannot be negated by
expressly repudiating it in the management contract and providing therein that the
"employee" is an independent contractor when the terms of the agreement clearly show
otherwise. For, the employment status of a person is defined and prescribed by law and not
by what the parties say it should be. In determining the status of the management contract,
the "four-fold test" on employment earlier mentioned has to be applied.[69] (Emphasis and
underscoring supplied)

2. Distinguishing radio and television talent as independent contractor from a


newscaster and co-anchor as an employee

Sonza v. ABS-CBN Broadcasting Corp., G. R. No. 138051, June 10, 2004


Same; Same; Same; Same; The greater the supervision and control the hirer exercises, the more
likely the worker is deemed an employee; The less control the hirer exercises, the more likely the worker
is considered an independent contractor; Applying the control test, SONZA is not an employee but an
independent contractor.—Applying the control test to the present case, we find that SONZA is not an
employee but an independent contractor. The control test is the most important test our courts apply in
distinguishing an employee from an independent contractor. This test is based on the extent of control the
hirer exercises over a worker. The greater the supervision and control of the hirer exercises, the more
likely the worker is deemed an employee. The converse holds true as well—the less control the hirer
exercises, the more likely the worker is considered an independent contractor.
Same; Same; Same; Same; ABS-CBN did not exercise control over the means and methods of
performance of SONZA’s work.—We find that ABS-CBN was not involved in the actual performance
that produced the finished product of SONZA’s work. ABS-CBN did not instruct SONZA how to
perform his job. ABS-CBN merely reserved the right to modify the program format and airtime schedule
“for more effective programming.” ABS-CBN’s sole concern was the quality of the shows and their
standing in the ratings. Clearly, ABS-CBN did not exercise control over the means and methods of
performance of SONZA’s work.

FACTS:

In May 1994, ABS-CBN signed an Agreement with the Mel and Jay Management and Development
Corporation (MJMDC). MJMDC agreed to provide Jay Sonza’s services exclusively to ABS-CBN as talent
for radio and television. ABS-CBN agreed to pay for Sonza's services a monthly talent fee of ₱310,000 for
the first year and ₱317,000 for the second and third year of the Agreement.

On April 1, 1996, Sonza wrote a letter to ABS-CBN President Eugenio Lopez III, accusing ABS-CBN of
violating the Agreement.

On April 30, 1996, Sonza filed a complaint before the Department of Labor and Employment (DOLE),
alleging that that ABS-CBN did not pay his salaries, separation pay, service incentive leave pay, 13th
month pay, signing bonus, travel allowance and amounts due under the Employees Stock Option Plan.

ABS-CBN filed a Motion to Dismiss on the ground that no employer-employee relationship existed
between the parties.

LABOR ARBITER: Dismissed Sonza's complaint for lack of jurisdiction, ruling that because Sonza is a
"talent," he cannot be considered an employee.

NLRC: Dismissed Sonza's Motion for Reconsideration.

COURT OF APPEALS: Affirmed NLRC ruling. The CA ruled that the allegations of Sonza against
ABS-CBN did not constitute a labor dispute because there was no employer-employee relationship to
begin with. If anything, Sonza's allegations constitute an action for breach of contractual obligation, which
is intrinsically a civil dispute to be resolved by a civil court, not the Labor Arbiter or the NLRC.

ISSUE:

Whether Jay Sonza was an employee of ABS-CBN? -- NO.


HELD:

The Supreme Court held that Sonza was not an employee of ABS-CBN. As a "talent," he was an
independent contractor. In coming up with this conclusion, the Court looked at the essential elements of
employer-employee relationship and applied the control test.

(a) Selection and engagement of employee


ABS-CBN engaged Sonza's services to co-host its television and radio programs because of his peculiar
skills, talent and celebrity status. These are indicative, but not conclusive, of an independent contractual
relationship

(b) Payment of wages


The Court held that whatever benefits Sonza enjoyed (SSS, Medicare, 13th month pay) arose from
contract and not because of an employer-employee relationship

(c) Power of dismissal


For violation of any provision of the Agreement, either party may terminate their relationship. Sonza failed
to show that ABS-CBN could terminate his services on grounds other than breach of contract, such as
retrenchment to prevent losses as provided under labor laws.

(d) Power of control


The control test is the most important test our courts apply in distinguishing an employee from an
independent contractor. This test is based on the extent of control the hirer exercises over a worker. The
greater the supervision and control the hirer exercises, the more likely the worker is deemed an
employee. The converse holds true as well – the less control the hirer exercises, the more likely the
worker is considered an independent contractor.

In Sonza's case, ABS-CBN did not exercise control over the means and methods of his work. The
Court found that ABS-CBN was not involved in the actual performance that produced the finished
product of Sonza's work.

Second, the fact that he was subjected to ABS-CBN's rules and standards of performance was not
determinative of control as it was under his contract that he " shall abide with the rules and standards of
performance covering talents of ABS-CBN."

Third, the "exclusivity" clause in the Agreement was not a form of control. In the broadcast
industry, exclusivity is not necessarily the same as control. The hiring of exclusive talents is a
widespread and accepted practice in the entertainment industry. This practice is not designed to
control the means and methods of work of the talent, but simply to protect the investment of the
broadcast station.

Finally, the Supreme Court held that the right of labor to security of tenure as guaranteed in the
Constitution arises only if there is an employer-employee relationship under labor laws. Not every
performance of services for a fee creates an employer-employee relationship.

To hold that every person who renders services to another for a fee is an employee - to give meaning to
the security of tenure clause - will lead to absurd results."

Thelma-Dumpit Murillo v. Court of Appeals, G. R. No. 164252, June 8, 2007

Same; Same; Employer-Employee Relationship; Elements.—In Manila Water Company, Inc. v. Pena,
434 SCRA 53 (2004), we said that the elements to determine the existence of an employment relationship
are: (a) the selection and engagement of the employee, (b) the payment of wages, (c) the power of
dismissal, and (d) the employer’s power to control. The most important element is the employer’s control
of the employee’s conduct, not only as to the result of the work to be done, but also as to the means and
methods to accomplish it.

Same; Same; Regular Employment; Regular status arises from either the nature of the work of the
employee or the duration of his employment.—Concerning regular employment, the law provides for two
kinds of employees, namely: (1) those who are engaged to perform activities which are usually necessary
or desirable in the usual business or trade of the employer; and (2) those who have rendered at least one
year of service, whether continuous or broken, with respect to the activity in which they are employed. In
other words, regular status arises from either the nature of work of the employee or the duration of his
employment. In Benares v. Pancho, 457 SCRA 652 (2005), we very succinctly said: . . . [T]he primary
standard for determining regular employment is the reasonable connection between the particular activity
performed by the employee vis-à-vis the usual trade or business of the employer. This connection can be
determined by considering the nature of the work performed and its relation to the scheme of the
particular business or trade in its entirety. If the employee has been performing the job for at least a year,
even if the performance is not continuous and merely intermittent, the law deems repeated and continuing
need for its performance as sufficient evidence of the necessity if not indispensability of that activity to
the business. Hence, the employment is considered regular, but only with respect to such activity and
while such activity exists.

Facts: On October 2, 1995, under talent contract no. NT95-1805, private respondent Associated
Broadcasting Company (ABC) hired petitioner Thelma Dumpit-Murillo as a newscaster and
co-anchor of Balitang-Balita, an early evening news program. The contract was for a period of 3
months. It was renewed under talent contract nos. NT95-1915, NT96-3002, NT98-4984, and
NT99-5649. In addition, petitioner’s services were engaged for the program “Live on Five.” On
September 30, 1999, after 4 years of repeated renewals, petitioner’s talent contract expired. Two
weeks after the expiration of the last contract, petitioner sent a letter to Mr. Jose Javier, Vice
President for news and public affairs of ABC, informing the latter that she was still interested in
renewing her contract subject to a salary increase, thereafter, petitioner stopped reporting for work.
On November 5, 1999 she wrote Mr. Javier another letter.

Issue: Whether or not the continuous renewal of petitioner’s talent contracts constitute regularity
in the employment status.

Held: Yes. An employer-employee relationship was created when the private respondents started to
merely renew the contracts repeatedly 15 times for 4 consecutive years.

Petitioner was a regular employee under contemplation of law. The practice of having fixed-term
contracts in the industry does not automatically make all talent contracts valid and compliant with
labor law. The assertion that a talent contract exists does not necessarily prevent a regular
employment status.

The elements to determine the existence of an employment relationship are: a.) The selection and
engagement of the employee; b.) The payment of wages; c.) The power of dismissal; and d.) The
employer’s control of the employee’s conduct, not only as to the result of the work to be done, but
also as to the means and methods to accomplish it.

The duties of petitioner as enumerated in her employment contract indicate that ABC had control
over the work or petitioner. Aside from control, ABC also dictated the work assignments and
payment of petitioner’s wages. ABC also had power to dismiss her. All these being present, clearly
there existed an employment relationship between petitioner and ABC.

Concerning regular employment, the law provides for 2 kinds of employees, namely: 1.) Those who
are engaged to perform activities which are usually necessary or desirable in the usual business or
trade of the employer; and 2.) Those who have rendered at least one year of service, whether
continuous or broken with respect to the activity in which they are employed. In other words,
regular status arises from either the nature of work of the employee or the duration of his
employment.

The primary standard of determining regular employment is the reasonable connection between the
particular activity performed by the employee vis-a-vis the usual trade or business of the employer.
This connection can be determined by considering the nature of the work performed and its relation
to the scheme of the particular business or trade in its entirety. If the employee has been performing
the job for at least a year, even if the performance is not continuous and merely intermittent, the law
deems repeated and continuing need for its performance as sufficient evidence of the necessity if
not indispensability of that activity to the business

3. Comparison of Murillo case from Sonza case


Sonza was engaged by ABS-CBN in view of his "unique skills, talent and celebrity status not
possessed by ordinary employees." His work was for radio and television programs.

On the other hand, Dumpit-Murillo was hired by ABC as a newscaster and co-anchor.

Sonza’s talent fee amounted to ₱317,000.00 per month, which this court found to be a substantial
amount that indicated he was an independent contractor rather than a regular employee.

Meanwhile, Dumpit-Murillo’s monthly salary was ₱28,000.00, a very low amount compared to what
Sonza received.

Sonza was unable to prove that ABS-CBN could terminate his services apart from breach of
contract. There was no indication that he could be terminated based on just or authorized causes
under the Labor Code. In addition, ABS-CBN continued to pay his talent fee under their
agreement, even though his programs were no longer broadcasted.

Dumpit-Murillo was found to have been illegally dismissed by her employer when they did not renew
her contract on her fourth year with ABC.

In Sonza, this court ruled that ABS-CBN did not control how Sonza delivered his lines, how he
appeared on television, or how he sounded on radio. All that Sonza needed was his talent. Further,
"ABS-CBN could not terminate or discipline SONZA even with the means and methods of
performance of his work . . . did not meet ABS-CBN’s approval."

In Dumpit-Murillo, the duties and responsibilities enumerated in her contract was a clear indication
that ABC had control over her work.

Fuji Television Network Inc. v. Espiritu, G. R. No. 204944-45, December 3, 2014


Same; Same; Independent Contractors; There is no employer-employee relationship between the
contractor and principal who engages the contractor’s services, but there is an employer-employee
relationship between the contractor and workers hired to accomplish the work for the principal.—In
Department Order No. 18-A, Series of 2011, of the Department of Labor and Employment, a contractor is
defined as having: Section 3. . . . . . . . (c) . . . an arrangement whereby a principal agrees to put out or
farm out with a contractor the performance or completion of a specific job, work or service within a
definite or predetermined period, regardless of whether such job, work or service is to be performed or
completed within or outside the premises of the principal. This department order also states that there is a
trilateral relationship in legitimate job contracting and subcontracting arrangements among the principal,
contractor, and employees of the contractor. There is no employer-employee relationship between the
contractor and principal who engages the contractor’s services, but there is an employer-employee
relationship between the contractor and workers hired to accomplish the work for the principal.
Same; Same; Same; Since no employer-employee relationship exists between independent
contractors and their principals, their contracts are governed by the Civil Code provisions on contracts
and other applicable laws.—Since no employer-employee relationship exists between independent
contractors and their principals, their contracts are governed by the Civil Code provisions on contracts
and other applicable laws. A contract is defined as “a meeting of minds between two persons whereby one
binds himself, with respect to the other, to give something or to render some service.” Parties are free to
stipulate on terms and conditions in contracts as long as these “are not contrary to law, morals, good
customs, public order, or public policy.” This presupposes that the parties to a contract are on equal
footing. They can bargain on terms and conditions until they are able to reach an agreement.
Same; Same; Same; Employees under fixed-term contracts cannot be independent contractors
because in fixed-term contracts, an employer-employee relationship exists.—Fuji’s argument that
Arlene was an independent contractor under a fixed-term contract is contradictory. Employees under
fixed-term contracts cannot be independent contractors because in fixed-term contracts, an
employer-employee relationship exists. The test in this kind of contract is not the necessity and
desirability of the employee’s activities, “but the day certain agreed upon by the parties for the
commencement and termination of the employment relationship.” For regular employees, the
necessity and desirability of their work in the usual course of the employer’s business are the determining
factors. On the other hand, independent contractors do not have employer-employee relationships with
their principals.
Same; Same; Same; Wages should not be the conclusive factor in determining whether one is an
employee or an independent contractor.—The Court of Appeals did not err when it relied on the ruling in
Dumpit-Murillo v. Court of Appeals, 524 SCRA 290 (2007), and affirmed the ruling of the National Labor
Relations Commission finding that Arlene was a regular employee. Arlene was hired by Fuji as a news
producer, but there was no showing that she was hired because of unique skills that would distinguish her
from ordinary employees. Neither was there any showing that she had a celebrity status. Her monthly
salary amounting to US$1,900.00 appears to be a substantial sum, especially if compared to her salary
when she was still connected with GMA. Indeed, wages may indicate whether one is an independent
contractor. Wages may also indicate that an employee is able to bargain with the employer for better pay.
However, wages should not be the conclusive factor in determining whether one is an employee or an
independent contractor.

Facts:

Arlene S. Espiritu ("Arlene") was engaged by Fuji Television Network, Inc. ("Fuji") as a news
correspondent/producer[4] "tasked to report Philippine news to Fuji through its Manila
Bureau field office."[5] Arlene's employment... contract initially provided for a term of one (1)
year but was successively renewed on a yearly basis with salary adjustment upon every
renewal.[6]

Arlene was diagnosed with lung cancer.[7] She informed Fuji about her condition. In turn,
the Chief of News Agency of Fuji, Yoshiki Aoki, informed Arlene "that the company will have
a problem renewing her contract"[8] since it would be difficult for her to perform her job.[9]
She "insisted that she was still fit to work as certified by her attending physician.

Arlene and Fuji signed a non-renewal contract... the day after Arlene signed the
non-renewal contract, she filed a complaint for illegal dismissal

She alleged that she was forced to sign the... non-renewal contract when Fuji came to know
of her illness and that Fuji withheld her salaries and other benefits

Labor Arbiter Corazon C. Borbolla dismissed Arlene's complaint... rlene appealed before
the National Labor Relations Commission.

the National Labor Relations Commission reversed the Labor Arbiter's decision.[21] It held
that Arlene was a regular employee with respect to the activities... for which she was
employed since she continuously rendered services that were deemed necessary and
desirable to Fuji's business.

n the assailed decision, the Court of Appeals affirmed the National Labor Relations
Commission with the modification that Fuji immediately reinstate Arlene to her position as
News Producer without loss of seniority rights,... eculia

Issues:

Whether the Court of Appeals correctly determined that no grave abuse of discretion was
committed by the National Labor Relations Commission when it ruled that Arlene was a
regular employee, not an independent contractor, and that she was illegally dismissed; and
Whether the Court of Appeals properly modified the National Labor Relations Commission's
decision by awarding reinstatement, damages, and attorney's fees

Ruling:

Whether the Court of Appeals correctly affirmed the National Labor Relations Commission's
finding that Arlene was a regular employee

Fuji alleges that Arlene was an independent contractor, citing Sonza v. ABS-CBN and
relying on the following facts: (1) she was hired because of her skills; (2) her salary was
US$1,900.00, which is higher than the normal rate; (3) she had the power to bargain with
her... employer; and (4) her contract was for a fixed term.

Arlene argues that she was a regular employee because Fuji had control and supervision
over her work. The news events that she covered were all based on the instructions of
Fuji.[142] She maintains that the successive renewal of her employment contracts for... four
(4) years indicates that her work was necessary and desirable.

On her illness, Arlene points out that it was not a ground for her dismissal because her
attending physician certified that she was fit to work.

Fuji's argument that Arlene was an independent contractor under a fixed-term contract is
contradictory. Employees under fixed-term contracts cannot be independent contractors
because in fixed-term contracts, an employer-employee relationship exists. The test in this
kind of... contract is not the necessity and desirability of the employee's activities, "but the
day certain agreed upon by the parties for the commencement and termination of the
employment relationship."[179] For regular employees, the necessity and desirability of...
their work in the usual course of the employer's business are the determining fac... tors. On
the other hand, independent contractors do not have employer-employee relationships with
their principals.

Arlene was hired by Fuji as a news producer, but there was no showing that she was hired
because of unique skills that would distinguish her from ordinary employees. Neither was
there any showing that she had a celebrity status. Her monthly salary amounting to
US$1,900.00 appears to be a substantial sum, especially if compared to her salary when
she was... still connected with GMA.[199] Indeed, wages may indicate whether one is an
independent contractor. Wages may also indicate that an employee is able to bargain with
the employer for better pay. However, wages should not be the conclusive factor in...
determining whether one is an employee or an independent contractor.

Fuji had the power to dismiss Arlene, as provided for in paragraph 5 of her professional
employment contract.[200] Her contract also indicated that Fuji had control over her work
because she was required to work for eight (8) hours from Monday to Friday,... although on
flexible time.[201] Sonza was not required to work for eight (8) hours, while Dumpit-Murillo
had to be in ABC to do both on-air and off-air tasks.

There is no evidence showing that Arlene was accorded due process. After informing her
employer of her lung cancer, she was not given the chance to present medical certificates.
Fuji immediately concluded that Arlene could no longer perform her duties because of
chemotherapy. It... did not ask her how her condition would affect her work. Neither did it
suggest for her to take a leave, even though she was entitled to sick leaves. Worse, it did
not present any certificate from a competent public health authority. What Fuji did was to
inform her that her... contract would no longer be renewed, and when she did not agree, her
salary was withheld.

well-entrenched is the rule that an illegally dismissed employee is entitled to reinstatement


as a matter of right. . . .

To protect labor's security of tenure, we emphasize that the doctrine of "strained


relations" should be strictly applied so as not to deprive an illegally dismissed employee of
his right to reinstatement. Every labor dispute almost always results in "strained relations"
and the phrase cannot be given an overarching interpretation, otherwise, an unjustly
dismissed employee can never be reinstated.[245] (Citations omitted)

The Court of Appeals reasoned that strained relations are a question of fact that must be
supported by evidence.[246] No evidence was presented by Fuji to prove that reinstatement
was no longer feasible. Fuji did not allege that it ceased operations or that

Arlene's position was no longer available. Nothing in the records shows that Arlene's
reinstatement would cause an atmosphere of antagonism in the workplace. Arlene filed her
complaint in 2009. Five (5) years are not yet a substantial period[247] to bar...
reinstatement.

Principles:

It is the burden of the employer to prove that a person whose services it pays for is an
independent contractor rather than a regular employee with or without a fixed term. That a
person has a disease does not per se entitle the employer to terminate his or her services.
Termination is the last resort. At the very least, a competent public health authority must
certify that the disease cannot be cured within six (6) months, even with appropriate
treatment.

A petition for certiorari under Rule 65 is an original action where the issue is limited to grave
abuse of discretion. As an original action, it cannot be considered as a continuation of the
proceedings of the labor tribunals.

On the other hand, a petition for review on certiorari under Rule 45 is a mode of appeal
where the issue is limited to questions of law. In labor cases, a Rule 45 petition is limited to
reviewing whether the Court of Appeals correctly determined the presence or absence of
grave... abuse of discretion and deciding other jurisdictional errors of the National Labor
Relations Commission.

Another classification of employees, i.e., employees with fixed-term contracts, was


recognized in Brent School, Inc. v. Zamora[150] where this court discussed that:

Logically, the decisive determinant in the term employment should not be the
activities that the employee is called upon to perform, but the day certain agreed
upon by the parties for the commencement and termination of their employment
relationship, a day certain being understood to be "that which must necessarily
come, although it may not be known when."[151] (Emphasis in the original)

GMA Network, Inc. v. Pabriga[154] expounded the doctrine on fixed-term contracts laid
down in Brent in the following manner:

Cognizant of the possibility of abuse in the utilization of fixed-term employment contracts,


we emphasized in Brent that where from the circumstances it is apparent that the periods
have been imposed to preclude acquisition of tenurial security by the employee, they...
should be struck down as contrary to public policy or morals. We thus laid down indications
or criteria under which "term employment" cannot be said to be in circumvention of the law
on security of tenure, namely:

1. The fixed period of employment was knowingly and voluntarily agreed upon by the
parties without any force, duress, or improper pressure being brought to bear upon
the employee and absent any other circumstances vitiating his consent; or
2. It satisfactorily appears that the employer and the employee dealt with each other on
more or less equal terms with no moral dominance exercised by the former or the
latter.

On the other hand, an independent contractor is defined as:

one who carries on a distinct and independent business and undertakes to


perform the job, work, or service on its own account and under one's own
responsibility according to one's own manner and method, free from the control and
direction of the principal in... all matters connected with the performance of the work
except as to the results thereof.

Orozco v. Court of Appeals, Wilhelmina Orozco was a columnist for the Philippine Daily
Inquirer. This court ruled that she was an independent contractor because of her "talent,
skill, experience, and her unique viewpoint as a feminist advocate." In addition, the
Philippine Daily Inquirer did not have the power of control over Orozco, and she worked at
her own pleasure.

Semblante v. Court of Appeals involved a masiador and a sentenciador. This court ruled
that "petitioners performed their functions as masiador and sentenciador free from the
direction and control of respondents" and that the masiador and sentenciador "relied mainly
on their 'expertise that is characteristic of the cockfight gambling.'" Hence, no
employer-employee relationship existed.

Bernarte v. Philippine Basketball Association[171] involved a basketball referee. This court


ruled that "a referee is an independent contractor, whose special skills and independent
judgment are required specifically for such position and cannot... possibly be controlled by
the hiring party."[172]

In these cases, the workers were found to be independent contractors because of their
unique skills and talents and the lack of control over the means and methods in the
performance of their work.

Since no employer-employee relationship exists between independent contractors and their


principals, their contracts are governed by the Civil Code provisions on contracts and other
applicable laws.

However, there may be a situation where an employee's work is necessary but is not
always desirable in the usual course of business of the employer. In this situation, there is
no regular employment.

San Miguel Corporation v. National Labor Relations Commission,[206] Francisco de


Guzman was hired to repair furnaces at San Miguel Corporation's Manila glass plant. He
had a separate contract for every furnace that he repaired. He filed a... complaint for illegal
dismissal three (3) years after the end of his last contract.[207] In ruling that de Guzman did
not attain the status of a regular employee, this court explained:

The process of manufacturing glass requires a furnace, which has a... limited operating life.
Petitioner resorted to hiring project or fixed term employees in having said furnaces repaired
since said activity is not regularly performed. Said furnaces are to be repaired or overhauled
only in case of need and after being used continuously for a... varying period of five (5) to
ten (10) years.

As stated in Price, et al. v. Innodata Corp., et al.:[228]

The employment status of a person is defined and prescribed by law and not by what the
parties say it should be. Equally important to consider is that a contract of employment is
impressed with public interest such that labor contracts must yield to the common good.

Thus, provisions of applicable statutes are deemed written into the contract, and the parties
are not at liberty to insulate themselves and their relationships from the impact of labor laws
and regulations by simply contracting with each other.

4. Independent contractor recognized by jurisprudence

Radio and television talent

Sonza v. ABS-CBN Broadcasting Corp., G. R. No. 138051, June 10, 2004, supra
Same; Same; Same; Same; The greater the supervision and control the hirer exercises, the more
likely the worker is deemed an employee; The less control the hirer exercises, the more likely the worker
is considered an independent contractor; Applying the control test, SONZA is not an employee but an
independent contractor.—Applying the control test to the present case, we find that SONZA is not an
employee but an independent contractor. The control test is the most important test our courts apply in
distinguishing an employee from an independent contractor. This test is based on the extent of control the
hirer exercises over a worker. The greater the supervision and control the hirer exercises, the more likely
the worker is deemed an employee. The converse holds true as well—the less control the hirer exercises,
the more likely the worker is considered an independent contractor.
Same; Same; Same; Same; ABS-CBN did not exercise control over the means and methods of
performance of SONZA’s work.—We find that ABS-CBN was not involved in the actual performance that
produced the finished product of SONZA’s work. ABS-CBN did not instruct SONZA how to perform his
job. ABS-CBN merely reserved the right to modify the program format and airtime schedule “for more
effective programming.” ABS-CBN’s sole concern was the quality of the shows and their standing in the
ratings. Clearly, ABS-CBN did not exercise control over the means and methods of performance of
SONZA’s work.
Same; Same; Same; Same; A radio broadcast specialist who works under minimal supervision is an
independent contractor.—A radio broadcast specialist who works under minimal supervision is an
independent contractor. SONZA’s work as television and radio program host required special skills and
talent, which SONZA admittedly possesses. The records do not show that ABS-CBN exercised any
supervision and control over how SONZA utilized his skills and talent in his shows.

Newspaper columnist

Orozco v. Court of Appeals, G. R. No. 155207, August 13, 2008


Same; Same; Same; Newspapers; Columnists; Reporters; The newspaper’s power to approve or
reject publication of any specific article a columnist writes for her column cannot be the control
contemplated in the “control test,” as it is but logical that one who commissions another to do a piece of
work should have the right to accept or reject the product; A regular reporter is not as independent in
doing his or her work for the newspaper.—The newspaper’s power to approve or reject publication of any
specific article she wrote for her column cannot be the control contemplated in the “control test,” as it is
but logical that one who commissions another to do a piece of work should have the right to accept or
reject the product. The important factor to consider in the “control test” is still the element of control over
how the work itself is done, not just the end result thereof. In contrast, a regular reporter is not as
independent in doing his or her work for the newspaper. We note the common practice in the newspaper
business of assigning its regular reporters to cover specific subjects, geographical locations, government
agencies, or areas of concern, more commonly referred to as “beats.” A reporter must produce stories
within his or her particular beat and cannot switch to another beat without permission from the editor. In
most newspapers also, a reporter must inform the editor about the story that he or she is working on for
the day. The story or article must also be submitted to the editor at a specified time. Moreover, the editor
can easily pull out a reporter from one beat and ask him or her to cover another beat, if the need arises.
Wilhelmina Orozco was hired as a writer by the Philippine Daily Inquirer (PDI) in
1990. She was the columnist of “Feminist Reflections” under the Lifestyle section of
the publication. She writes on a weekly basis and on a per article basis
(P250-300/article).

In 1991, Magsanoc as the editor-in-chief sought to improve the Lifestyle section of


the paper. She said there were too many Lifestyle writers and that it was time to
reduce the number of writers. Orozco’s column was eventually dropped.

Orozco filed for a case for Illegal Dismissal against PDI and Magsanoc. Orozco won
in the Labor Arbiter. The LA ruled that there exists an employer-employee
relationship between PDI and Orozco hence Orozco is entitled to receive backwages,
reinstatement, and 13th month pay.

PDI appealed to the National Labor Relations Commission. The NLRC denied the
appeal because of the failure of PDI to post a surety bond as required by Article 223
of the Labor Code. The Court of Appeals reversed the NLRC.
ISSUE: Whether or not there exists an employer-employee relationship between PDI
and Orozco. Whether or not PDI’s appeal will prosper.

HELD: Under Article 223 of the Labor Code:


ART. 223. Appeal. – Decisions, awards or orders of the Labor Arbiter are final and
executory unless appealed to the Commission by any or both parties within ten (10)
calendar days from receipt of such decisions, awards, or orders.
In case of a judgment involving a monetary award, an appeal by the employer may be
perfected only upon the posting of a cash or surety bond issued by a reputable
bonding company duly accredited by the Commission in the amount equivalent to the
monetary award in the judgment appealed from.

The requirement that the employer post a cash or surety bond to perfect its/his appeal
is apparently intended to assure the workers that if they prevail in the case, they will
receive the money judgment in their favor upon the dismissal of the employer’s
appeal. It was intended to discourage employers from using an appeal to delay, or
even evade, their obligation to satisfy their employees’ just and lawful claims.

But in this case, this principle is relaxed by the Supreme Court considering the fact
that the Labor Arbiter, in ruling that the Orozco is entitled to backwages, did not
provide any computation.

The case is then remanded to the Labor Arbiter for the computation. This necessarily
pended the resolution of the other issue of whether or not there exists an
employer-employee relationship between PDI and Orozco.

Masiador and sentenciador in cockfight gambling

Semblante v. Court of Appeals, G. R. No. 196426, August 15, 2011


Same; Same; Same; Same; Employer-Employee Relationship; Four-Fold Test of Employment;
Cockfighting; Masiadors and Sentenciadors; The rule on the posting of an appeal bond cannot defeat the
substantive rights of respondents to be free from an unwarranted burden of answering for an illegal
dismissal for which they were never responsible; Since the complainants performed their functions as
masiador and sentenciador free from the direction and control of respondents, and that in the conduct of
their work, they relied mainly on their “expertise that is characteristic of the cockfight gambling,” and
were never given by respondents any tool needed for the performance of their work, they are not
considered as employees of the cockpit operator.—While respondents had failed to post their bond within
the 10-day period provided above, it is evident, on the other hand, that petitioners are NOT employees of
respondents, since their relationship fails to pass muster the four-fold test of employment We have
repeatedly mentioned in countless decisions: (1) the selection and engagement of the employee; (2) the
payment of wages; (3) the power of dismissal; and (4) the power to control the employee’s conduct, which
is the most important element. As found by both the NLRC and the CA, respondents had no part in
petitioners’ selection and management; petitioners’ compensation was paid out of the arriba (which is a
percentage deducted from the total bets), not by petitioners; and petitioners performed their functions as
masiador and sentenciador free from the direction and control of respondents. In the conduct of their
work, petitioners relied mainly on their “expertise that is characteristic of the cockfight gambling,” and
were never given by respondents any tool needed for the performance of their work. Respondents, not
being petitioners’ employers, could never have dismissed, legally or illegally, petitioners, since
respondents were without power or prerogative to do so in the first place. The rule on the posting of an
appeal bond cannot defeat the substantive rights of respondents to be free from an unwarranted burden of
answering for an illegal dismissal for which they were never responsible.
FACTS: Petitioners Marticio Semblante (Semblante) and Dubrick Pilar
(Pilar) assert that they were hired by respondents-spouses Vicente and
Maria Luisa Loot, the owners of Gallera de Mandaue (the cockpit), as the
official masiador and sentenciador, respectively, of the cockpit sometime
in 1993.

As themasiador, Semblante calls and takes the bets from the gamecock
owners and other bettors and orders the start of the cockfight. He also
distributes the winnings after deducting thearriba, or the commission for
the cockpit. Meanwhile, as the sentenciador, Pilar oversees the proper
gaffing of fighting cocks, determines the fighting cocks physical
condition and capabilities to continue the cockfight, and eventually
declares the result of the cockfight.

On November 14, 2003, however, petitioners were denied entry into the
cockpit upon the instructions of respondents, and were informed of the
termination of their services effective that date. This prompted
petitioners to file a complaint for illegal dismissal against respondents.

In answer, respondents denied that petitioners were their employees and


alleged that they were associates of respondents independent contractor,
Tomas Vega. Respondents claimed that petitioners have no regular
working time or day and they are free to decide for themselves whether
to report for work or not on any cockfighting day. In times when there
are few cockfights inGallera de Mandaue, petitioners go to other cockpits
in the vicinity. Lastly, petitioners, so respondents assert, were only
issued identification cards to indicate that they were free from the
normal entrance fee and to differentiate them from the general public.

Labor Arbiter Julie C. Rendoque found petitioners to be regular


employees of respondents as they performed work that was necessary
and indispensable to the usual trade or business of respondents for a
number of years. The Labor Arbiter also ruled that petitioners were
illegally dismissed, and so ordered respondents to pay petitioners their
backwages and separation pay.

Respondents counsel received the Labor Arbiters Decision on September


14, 2004. And within the 10-day appeal period, he filed the respondents
appeal with the NLRC on September 24, 2004, but without posting a
cash or surety bond equivalent to the monetary award granted by the
Labor Arbiter. It was only on October 11, 2004 that respondents filed an
appeal bond dated October 6, 2004. Hence, in a Resolution dated August
25, 2005, the NLRC denied the appeal for its non-perfection.
Subsequently, however, the NLRC, acting on respondents Motion for
Reconsideration, reversed its Resolution on the postulate that their
appeal was meritorious and the filing of an appeal bond, albeit belated, is
a substantial compliance with the rules.The NLRC held in its Resolution
of October 18, 2006 that there was no employer-employee relationship
between petitioners and respondents, respondents having no part in the
selection and engagement of petitioners, and that no separate individual
contract with respondents was ever executed by petitioners.

The appellate court found for respondents, noting that referees and
bet-takers in a cockfight need to have the kind of expertise that is
characteristic of the game to interpret messages conveyed by mere
gestures. Hence, petitioners are akin to independent contractors who
possess unique skills, expertise, and talent to distinguish them from
ordinary employees.

The CA refused to reconsider its Decision. Hence, petitioners came to


this Court, arguing in the main that the CA committed a reversible error
in entertaining an appeal, which was not perfected in the first place.

ISSUE: Did the CA err in entertaining an appeal which was not


perfected?

HELD: Indeed, the posting of a bond is indispensable to the perfection


of an appeal in cases involving monetary awards from the Decision of the
Labor Arbiter. Article 223 of the Labor Code provides:

Article 223. Appeal. Decisions, awards, or orders of the Labor Arbiter are
final and executory unless appealed to the Commission by any or both
partieswithin ten (10) calendar days from receipt of such decisions,
awards, or orders.Such appeal may be entertained only on any of the
following grounds:
In case of a judgment involving a monetary award,an appeal by the
employer may be perfected only upon the posting of a cash or surety
bondissued by a reputable bonding company duly accredited by the
Commission in the amount equivalent to the monetary award in the
judgment appealed from.

Time and again, however, this Court, considering the substantial merits
of the case, has relaxed this rule on, and excused the late posting of, the
appeal bond when there are strong and compelling reasons for the
liberality, such as the prevention of miscarriage of justice extant in the
caseor the special circumstances in the case combined with its legal
merits or the amount and the issue involved.After all, technical rules
cannot prevent courts from exercising their duties to determine and
settle, equitably and completely, the rights and obligations of the parties.
This is one case where the exception to the general rule lies.

While respondents had failed to post their bond within the 10-day period
provided above, it is evident, on the other hand, that petitioners are NOT
employees of respondents, since their relationship fails to pass muster
the four-fold test of employment We have repeatedly mentioned in
countless decisions:

(1) the selection and engagement of the employee;


(2) the payment of wages;
(3) the power of dismissal; and
(4) the power to control the employees conduct, which is the most
important element.

As found by both the NLRC and the CA, respondents had no part in
petitioners selection and management;petitioners compensation was
paid out of the arriba (which is a percentage deducted from the total
bets), not by petitioners;and petitioners performed their functions as
masiador and sentenciador from the direction and control of
respondents. In the conduct of their work, petitioners relied mainly on
their expertise that is characteristic of the cockfight gambling, and were
never given by respondents any tool needed for the performance of their
work.

Respondents, not being petitioners employers, could never have


dismissed, legally or illegally, petitioners, since respondents were
without power or prerogative to do so in the first place. The rule on the
posting of an appeal bond cannot defeat the substantive rights of
respondents to be free from an unwarranted burden of answering for an
illegal dismissal for which they were never responsible.

Strict implementation of the rules on appeals must give way to the


factual and legal reality that is evident from the records of this case.After
all, the primary objective of our laws is to dispense justice and equity, not
the contrary. DENIED.

Basketball referee

Bernate v. Philippine Basketball Association, G. R. No. 192084, September 14, 2011


Same; Same; The very nature of petitioner’s job of officiating a professional basketball game undoubtedly
calls for freedom of control by respondents.—With respondents that once in the playing court, the referees
exercise their own independent judgment, based on the rules of the game, as to when and how a call or
decision is to be made. The referees decide whether an infraction was committed, and the PBA cannot
overrule them once the decision is made on the playing court. The referees are the only, absolute, and
final authority on the playing court. Respondents or any of the PBA officers cannot and do not determine
which calls to make or not to make and cannot control the referee when he blows the whistle because such
authority exclusively belongs to the referees. The very nature of petitioner’s job of officiating a
professional basketball game undoubtedly calls for freedom of control by respondents.

Same; Same; Applicable foreign case law declares that a referee is an independent contractor, whose
special skills and independent judgment are required specifically for such position and cannot possibly be
controlled by the hiring party.—The applicable foreign case law declares that a referee is an independent
contractor, whose special skills and independent judgment are required specifically for such position and
cannot possibly be controlled by the hiring party.

FACTS:

Complainants Jose Mel Bernarte and Renato Guevarra aver that they were
invited to join the PBA as referees and were made to sign contracts on
year-to-year basis. However during the term of a new commissioner, Eala,
changes were made on the terms of their employment.

On January 15, 2004, Bernarte received a letter from the Commissioner


advising him that his contract would not be renewed citing his unsatisfactory
performance on and off the court. It was a total shock for Bernarte who was
awarded Referee of the year in 2003. He felt that the dismissal was caused by
his refusal to fix a game.

On the other hand, beginning 2002, complainant Guevarra signed a yearly


contract as Regular Class C referee. On May 6, 2003, respondent Martinez
issued a memorandum to Guevarra expressing dissatisfaction over his
questioning on the assignment of referees officiating out-of-town games.
Beginning February 2004, he was no longer made to sign a contract.

Respondents aver that complainants were not illegally dismissed because they
were not employees of the PBA. According to them, complainants entered into
2 contracts of retainer with the PBA in the year 2003 for the period of Jan. 1,
2003 to July 15, 2003 and September 1 to December 2003. After the lapse of the
second contract, PBA decided not to renew their contracts.

ISSUE:

Whether or not petitioner is an employee of respondent and has been illegally


dismissed.

RULING:

In determining the existence of an employer-employee relationship, case law


has consistently applied the four-fold test: (a) the selection and engagement
of the employee; (b) the payment of wages; (c) the power of dismissal; and (d)
the employer’s power to control the employee on the means and methods by
which the work is accomplished. The most important indicator of the presence
or absence of an employer-employee relationship is the so-called “control
test.”

The very nature of petitioner’s job of officiating a professional basketball game


undoubtedly calls for freedom of control by respondents. Once in the playing
court, the referees exercise their own independent judgment, based on the
rules of the game, as to when and how a call or decision is to be made. The
referees decided whether an infraction was committed, and the PBA cannot
overrule them once the decision is made on the playing court. The referees are
the only, absolute, and final authority on the playing court. Respondents or any
of the PBA officers cannot and do not determine which calls to make or not to
make and cannot control the referee when he blows the whistle because such
authority exclusively belongs to the referees.

Applicable foreign case law also declares that a referee is an independent


contractor, whose special skills and independent judgment are required
specifically for such position and cannot possibly be controlled by the hiring
party.

5. Insurance agents

Insurance agent as independent contractor and not an employee

Carungcung v. NLRC, Sun Life Assurance Co. of Canada, G.R. No. 118086. December 15,
1997

Labor Law; Employer-Employee Relationship; Independent Contractors; Insurance Agents; A person


whose stated annual income from her occupation is impressive by any standards—“in excess of
P3,000,000.00,” exclusive of overriding commissions—is not an ordinary run-of-the-mill employee, nor
even an average managerial employee or supervisor, and, by no means may be considered as dealing, or
having dealt, with her employer from an inferior position, as a disadvantaged, morally-dominated person
and can impel concurrence with the conclusion that he or she is an independent contractor, not an
employee.—It is germane to advert to the fact, which should by now be apparent, that Carungcong was
not your ordinary run-of-the-mill employee, nor even your average managerial employee or supervisor.
Her stated annual income from her occupation is impressive by any standards: “in excess of
P3,000,000.00,” exclusive of overriding commissions. Certainly, she may not be likened to an ordinary
person applying for employment, or an ordinary employee striving to keep his job, under the moral
dominance of the hiring entity or individual. By no means may Carungcong be considered as dealing, or
having dealt, with Sun Life from an inferior position, as a disadvantaged, morally-dominated person. She
must be deemed as having transacted with Sun Life’s executives on more or less equal terms. These
considerations impel concurrence with the conclusions of the challenged decision and resolution of
respondent Commission which considered Carungcong an independent contractor, not an employee of
Sun Life. It is significant that this issue of the precise status of Carungcong as an independent contractor,
evidently deemed decisive by respondent Commission, was discussed by it at some length not once, but
twice, first in its Decision of July 29, 1994, and then in its second Decision of October 28, 1994 resolving
the separate motions for reconsideration of the parties.

Facts:

Susan Carungcong is an “agent to solicit applications for insurance and annuity policies”
of Sun Life Assurance Company of Canada. Later, Carungcong was named New
Business Manager but per agreement, it was emphasized that she shall be considered
an independent contractor and not an employee. Some time later, the Internal Audit
Department discovered some anomalies in the special fund availments of Carungcong,
leading to an inquiry and her subsequent termination. Carungcong, therefore, instituted
proceedings to vindicate her case. The favorable decision of the Labor Arbiter was,
however, reversed later by the NLRC, stating that no employment relationship existed
between Carungcong and Sun Life.

Issue:
● W/N there is an employer-employee relationship

Held:

…It is true that complainant Carungcong’s duties and functions derived from then
existing agreements/contracts were made subject to rules and regulations issued by
respondent company, and for that matter, have likewise been made subject of certain
limitations imposed by said respondent company. Nonetheless, these are not sufficient
to accord the effect of establishing employer-employee relationship absent in this case.
This is so because the insurance business is not just any other ordinary business. It is
one that is imbued with public interest hence, it must be governed by the rules and
regulations of the state. The controls adverted to by complainant are latent in the kind of
business she is into and are mainly aimed at promoting the results the parties so desire
and do not necessarily create any employer-employee relationships, where the
employers’ controls have to interfere in the methods and means by which the employee
would like to employ to arrive at the desired results.

This is not without any jurisprudential support as earlier pointed out by herein
respondent. The Supreme Court in the case of Insular Life Assurance Co., Ltd. versus
National Labor Relations Commission and Melencio Basiao (179 SCRA 459)
emphatically discoursed in this wise:

Logically, the line should be drawn between rules that merely serve as guidelines
towards the achievement of the mutually desired result without dictating the means or
methods to be employed in attaining it, and those that control to fix the methodology
and bind or restrict the party hired to the use of such means. The first, which aim only to
promote the result, create no employer-employee relationship unlike the second, which
address both the result and the means used to achieve it. The distinction acquires
particular relevance in the case of an enterprise affected with public interest and is on
that account subject to regulation by the State with respect, not only to the relations
between insurer and insured but also to the internal affairs of the Insurance company.
Rules and regulations governing the conduct of the business are provided for in the
Insurance Code and enforced by the Insurance Commissioner. It is therefore usual and
expected for an insurance company to promulgate a set of rules to guide its commission
agents in selling its policies that they may not run afoul of the law and what it requires or
prohibits.

For that matter, complainant Carungcong was never paid a fixed wage or salary but was
mainly paid by commissions, depending on the level and volume of her
performance/production, the number of trained agents, when taken in and assigned to
her, being responsible for her added income as she gets a certain percentage from the
said agents' production as part of her commission.

Not every form of control may established employer employee relationship

Insular Life Assurance Co., Ltd. v. NLRC, Basiao, G.R. No. 84484 November 15, 1989
Labor Relations; Employer-Employee Relationship; Independent Contractor; Control Test; Not
every form of control over the conduct of the party hired in relation to the service rendered establishes
employer-employee relationship.—It is true that the “control test” expressed in the following
pronouncement of the Court in the 1956 case of Viana vs. Alejo Al-Lagadan: “x x x In determining the
existence of employer-employee relationship, the following elements are generally considered, namely:
(1) the selection and engagement of the employee; (2) the payment of wages; (3) the power of dismissal;
and (4) the power to control the employee conduct—although the latter is the most important element (35
Am. Jur. 445). x x x,” has been followed and applied in later cases, some fairly recent. Indeed, it is
without question a valid test of the character of a contract or agreement to render service. It should,
however, be obvious that not every form of control that the hiring party reserves to himself over the
conduct of the party hired in relation to the services rendered may be accorded the effect of establishing
an employer-employee relationship between them in the legal or technical sense of the term. A line must
be drawn somewhere, if the recognized distinction between an employee and an individual contractor is
not to vanish altogether.
Same; Same; Same; Same; Same; When an insurance agent is free to adopt his own selling methods
or is free to sell insurance at his own time, he is an independent contractor.—Logically, the line should be
drawn between rules that merely serve as guidelines towards the achievement of the mutually desired
result without dictating the means or methods to be employed in attaining it, and those that control or fix
the methodology and bind or restrict the party hired to the use of such means. The first, which aim only to
promote the result, create no employer-employee relationship unlike the second, which address both the
result and the means used to achieve it. The distinction acquires particular relevance in the case of an
enterprise affected with public interest, as is the business of insurance, and is on that account subject to
regulation by the State with respect, not only to the relations between insurer and insured but also to the
internal affairs of the insurance company. Rules and regulations governing the conduct of the business are
provided for in the Insurance Code and enforced by the Insurance Commissioner. It is, therefore, usual
and expected for an insurance company to promulgate a set of rules to guide its commission agents in
selling its policies that they may not run afoul of the law and what it requires or prohibits. Of such a
character are the rules which prescribe the qualifications of persons who may be insured, subject
insurance applications to processing and approval by the Company, and also reserve to the Company the
determination of the premiums to be paid and the schedules of payment. None of these really invades the
agent’s contractual prerogative to adopt his own selling methods or to sell insurance at his own time and
convenience, hence cannot justifiably be said to establish an employer-employee relationship between
him and the Company.
Facts:

Petitioner Insular Life entered into a contract with respondent Basiao where the latter is
authorized to solicit for insurance policies. Sometime later, the parties entered into
another contract which caused Basiao to organize an agency in order to fulfill its terms.
The contract being subsequently terminated by petitioner, Basiao sued the latter which
prompted also for the termination of their engagement under the first contract. Basiao
thus filed before the Ministry of Labor seeking to recover alleged unpaid commissions.
Petitioner contends that Basiao is not an employee but an independent contractor for
which they have no obligation to pay said commissions. The Labor Arbiter found for
Basiao ruling that there exists employer-employee relationship between him and
petitioner. NLRC affirmed.

Issue:

Whether or not employer-employee relationship existed between petitioner and Basiao.

Ruling: NO.

In determining the existence of employer-employee relationship, the following elements


are generally considered, namely: (1) the selection and engagement of the employee;
(2) the payment of wages; (3) the power of dismissal; and (4) the power to control the
employees’ conduct — although the latter is the most important element. It should,
however, be obvious that not every form of control that the hiring party reserves to
himself over the conduct of the party hired in relation to the services rendered may be
accorded the effect of establishing an employer-employee relationship between them in
the legal or technical sense of the term.

Rules and regulations governing the conduct of the business are provided for in the
Insurance Code and enforced by the Insurance Commissioner. It is, therefore, usual
and expected for an insurance company to promulgate a set of rules to guide its
commission agents in selling its policies that they may not run afoul of the law and what
it requires or prohibits. None of these really invades the agent’s contractual prerogative
to adopt his own selling methods or to sell insurance at his own time and convenience,
hence cannot justifiably be said to establish an employer-employee relationship
between him and the company.

The Court, therefore, rules that under the contract invoked by him, Basiao was not an
employee of the petitioner, but a commission agent, an independent contractor whose
claim for unpaid commissions should have been litigated in an ordinary civil action.

*In contrast to the case decided by the Court 10 years after, Insular Life
Assurance v. NLRC and Pantaleon De Los Reyes.

Royal Homes Marketing Corporation v. Alcantara, G. R. No. 195190, July 28, 2014
Same; Same; As long as the level of control does not interfere with the means and methods of
accomplishing the assigned tasks, the rules imposed by the hiring party on the hired party do not amount
to the labor law concept of control that is indicative of employer-employee relationship.—Not every form
of control is indicative of employer-employee relationship. A person who performs work for another and
is subjected to its rules, regulations, and code of ethics does not necessarily become an employee. As long
as the level of control does not interfere with the means and methods of accomplishing the assigned tasks,
the rules imposed by the hiring party on the hired party do not amount to the labor law concept of control
that is indicative of employer-employee relationship. In Insular Life Assurance Co., Ltd. v. National
Labor Relations Commission, 179 SCRA 459 (1989), it was pronounced that: Logically, the line should
be drawn between rules that merely serve as guidelines towards the achievement of the mutually desired
result without dictating the means or methods to be employed in attaining it, and those that control or fix
the methodology and bind or restrict the party hired to the use of such means. The first, which aim only to
promote the result, create no employer-employee relationship unlike the second, which address both the
result and the means used to achieve it.
Facts:

In 1994, Royale Homes, a corporation engaged in marketing real estates, appointed


Alcantara as its Marketing Director for a fixed period of one year. His work consisted mainly
of marketing Royale Homes' real estate inventories on an exclusive basis. Royale Homes...
reappointed him for several consecutive years, the last of which covered the period January
1 to December 31, 2003 where he held the position of Division 5 Vice-President-Sales.

On December 17, 2003, Alcantara filed a Complaint for Illegal Dismissal[9] against Royale
Homes

Alcantara alleged that he is a regular employee of Royale Homes since he is performing


tasks that are necessary and desirable to its business; that in 2003 the company gave him
P1.2 million for the services he rendered to it; that in the first week of November 2003,...
however, the executive officers of Royale Homes told him that they were wondering why he
still had the gall to come to office and sit at his table;[10] and that the acts of the executive
officers of Royale Homes amounted to his dismissal from work without... any valid or just
cause and in gross disregard of the proper procedure for dismissing employees. Thus, he
also impleaded the corporate officers who, he averred, effected his dismissal in bad faith
and in an oppressive manner.

Royale Homes, on the other hand, vehemently denied that Alcantara is its employee. It
argued that the appointment paper of Alcantara is clear that it engaged his services as an
independent sales contractor for a fixed term of one year only. He never received any
salary,... 13th month pay, overtime pay or holiday pay from Royale Homes as he was paid
purely on commission basis. In addition, Royale Homes had no control on how Alcantara
would accomplish his tasks and responsibilities as he was free to solicit sales at any time
and by any manner which... he may deem appropriate and necessary. He is even free to
recruit his own sales personnel to assist him in pursuance of his sales target.
Issues:

whether Alcantara was an independent contractor or an employee of Royale Homes.

Ruling:

The Petition is impressed with merit.

The primary evidence of the nature of the parties' relationship in this case is the written
contract that they signed and executed in pursuance of their mutual agreement. While the
existence of employer-employee relationship is a matter of law, the characterization made
by... the parties in their contract as to the nature of their juridical relationship cannot be
simply ignored, particularly in this case where the parties' written contract unequivocally
states their intention at the time they entered into it.

In this case, the contract,[27] duly signed and not disputed by the parties, conspicuously
provides that "no employer-employee relationship exists between" Royale Homes and
Alcantara, as well as his sales agents. It is clear that they did not want to... be bound by
employer-employee relationship at the time of the signing of the contract.

Since "the terms of the contract are clear and leave no doubt upon the intention of the
contracting parties, the literal meaning of its stipulations should control."[28] No
construction is even needed as they already expressly state their... intention. Also, this
Court adopts the observation of the NLRC that it is rather strange on the part of Alcantara,
an educated man and a veteran sales broker who claimed to be receiving P1.2 million as
his annual salary, not to have contested the portion of the contract... expressly indicating
that he is not an employee of Royale Homes if their true intention were otherwise.

Not every form of control is indicative of employer-employee relationship. A person who


performs work for another and is subjected to its rules, regulations, and code of ethics does
not necessarily become an employee.[34] As long as the level of... control does not
interfere with the means and methods of accomplishing the assigned tasks, the rules
imposed by the hiring party on the hired party do not amount to the labor law concept of
control that is indicative of employer-employee relationship.

In this case, the Court agrees with Royale Homes that the rules, regulations, code of ethics,
and periodic evaluation alluded to by Alcantara do not involve control over the means and
methods by which he was to perform his job. Understandably, Royale Homes has to fix
the... price, impose requirements on prospective buyers, and lay down the terms and
conditions of the sale, including the mode of payment, which the independent contractors
must follow. It is also necessary for Royale Homes to allocate its inventories among its
independent... contractors, determine who has priority in selling the same, grant
commission or allowance based on predetermined criteria, and regularly monitor the result
of their marketing and sales efforts. But to the mind of this Court, these do not pertain to
the means and methods... of how Alcantara was to perform and accomplish his task of
soliciting sales. They do not dictate upon him the details of how he would solicit sales or
the manner as to how he would transact business with prospective clients. In Tongko, this
Court held that... guidelines or rules and regulations that do not pertain to the means or
methods to be employed in attaining the result are not indicative of control as understood in
labor law.

Guidelines indicative of labor law "control," as the first Insular Life case tells us, should not
merely relate to the mutually desirable... result intended by the contractual relationship; they
must have the nature of dictating the means or methods to be employed in attaining the
result, or of fixing the methodology and of binding or restricting the party hired to the use of
these means.

As the party claiming the existence of employer-employee relationship, it behoved upon


Alcantara to prove the elements thereof, particularly Royale Homes' power of control over
the means and methods of accomplishing the work.[38] He, however, failed... to cite
specific rules, regulations or codes of ethics that supposedly imposed control on his means
and methods of soliciting sales and dealing with prospective clients. On the other hand, this
case is replete with instances that negate the element of control and the... existence of
employer-employee relationship. Notably, Alcantara was not required to observe definite
working hours.[39] Except for soliciting sales, Royale Homes did not assign other tasks to
him. He had full control over the means and... methods of accomplishing his tasks as he
can "solicit sales at any time and by any manner which [he may] deem appropriate and
necessary." He performed his tasks on his own account free from the control and direction
of Royale Homes in all matters connected therewith,... except as to the results thereof.[40]

Neither does the repeated hiring of Alcantara prove the existence of employer-employee
relationship.[41] As discussed above, the absence of control over the means and methods
disproves employer-employee relationship. The continuous rehiring of

Alcantara simply signifies the renewal of his contract with Royale Homes, and highlights his
satisfactory services warranting the renewal of such contract. Nor does the exclusivity
clause of contract establish the existence of the labor law concept of control.

The element of payment of wages is also absent in this case. As provided in the contract,
Alcantara's remunerations consist only of commission override of 0.5%, budget allocation,
sales incentive and other forms of company support. There is no proof that he received
fixed... monthly salary. No payslip or payroll was ever presented and there is no proof that
Royale Homes deducted from his supposed salary withholding tax or that it registered him
with the Social Security System, Philippine Health Insurance Corporation, or Pag-Ibig Fund.

All of these indicate an independent contractual relationship.

This Court is, therefore, convinced that Alcantara is not an employee of Royale Homes, but
a mere independent contractor.

Principles:

Not every form of control that a hiring party imposes on the hired party is indicative of
employee-employer relationship. Rules and regulations that merely serve as guidelines
towards the achievement of a mutually desired result without dictating the means... and
methods of accomplishing it do not establish employer-employee relationship.

To be sure, the Agreement's legal characterization of the nature of the relationship cannot
be conclusive and binding on the courts; x x x the characterization of the juridical
relationship the Agreement embodied is a matter of law that is for the courts to... determine.
At the same time, though, the characterization the parties gave to their relationship in the
Agreement cannot simply be brushed aside because it embodies their intent at the time
they entered the Agreement, and they were governed by this understanding throughout...
their relationship. At the very least, the provision on the absence of employer-employee
relationship between the parties can be an aid in considering the Agreement and its
implementation, and in appreciating the other evidence on record.

Guidelines indicative of labor law "control," as the first Insular Life case tells us, should not
merely relate to the mutually desirable... result intended by the contractual relationship; they
must have the nature of dictating the means or methods to be employed in attaining the
result, or of fixing the methodology and of binding or restricting the party hired to the use of
these means.
Exclusivity

AFP Mutual Benefit Association, Inc. v. National Labor Relations Commission, G.R. No.
102199, January 28, 1997
Same; Same; Control Test; Insurance; Agency; Insurance Agents; The fact that an insurance
underwriter was required to solicit business exclusively for a mutual benefit association could hardly be
considered as control in labor jurisprudence.—The difficulty lies in correctly assessing if certain factors
or elements properly indicate the presence of control. Anent the issue of exclusivity in the case at bar, the
fact that private respondent was required to solicit business exclusively for petitioner could hardly be
considered as control in labor jurisprudence. Under Memo Circulars No. 2–81 and 2–85, dated
December 17, 1981 and August 7, 1985, respectively, issued by the Insurance Commissioner, insurance
agents are barred from serving more than one insurance company, in order to protect the public and to
enable insurance companies to exercise exclusive supervision over their agents in their solicitation work.
Thus, the exclusivity restriction clearly springs from a regulation issued by the Insurance Commission,
and not from an intention by petitioner to establish control over the method and manner by which private
respondent shall accomplish his work. This feature is not meant to change the nature of the relationship
between the parties, nor does it necessarily imbue such relationship with the quality of control envisioned
by the law.

Same; Same; Same; Independent Contractors; The test to determine the existence of independent
contractorship is whether one claiming to be an independent contractor has contracted to do the work
according to his own methods and without being subject to the control of the employer except only as to
the result of the work.—To restate, the significant factor in determining the relationship of the parties is
the presence or absence of supervisory authority to control the method and the details of performance of
the service being rendered, and the degree to which the principal may intervene to exercise such control.
The presence of such power of control is indicative of an employment relationship, while absence thereof
is indicative of independent contractorship. In other words, the test to determine the existence of
independent contractorship is whether one claiming to be an independent contractor has contracted to do
the work according to his own methods and without being subject to the control of the employer except
only as to the result of the work. Such is exactly the nature of the relationship between petitioner and
private respondent.
Same; Same; Same.—Further, not every form of control that a party reserves to himself over the
conduct of the other party in relation to the services being rendered may be accorded the effect of
establishing an employer-employee relationship.
Facts:

Private respondent Eutiquio Bustamante was an insurance underwriter of petitioner AFP


Mutual Benefit Association until he was dismissed for misrepresentation and for
simultaneously selling insurance for another life insurance company in violation of their
agreement. Respondent signed a quitclaim after receiving his commissions but later on
discovered that he was entitled to more than the received amount. Thus, he filed a
complaint with the Office of the Insurance Commissioner but was advised to file before
the Department of Labor, which then ruled in his favor citing that employer-employee
relationship exists between him and petitioner. NLRC tribunal affirmed.

Issue:

Whether or not employer-employee relationship existed between the parties.

Ruling: NO.

The significant factor in determining the relationship of the parties is the presence or
absence of supervisory authority to control the method and the details of performance of
the service being rendered, and the degree to which the principal may intervene to
exercise such control. The presence of such power of control is indicative of an
employment relationship, while absence thereof is indicative of independent
contractorship. In other words, the test to determine the existence of independent
contractorship is whether one claiming to be an independent contractor has contracted
to do the work according to his own methods and without being subject to the control of
the employer except only as to the result of the work. Such is exactly the nature of the
relationship between petitioner and private respondent.

Private respondent was free to sell insurance at any time as he was not subject to
definite hours or conditions of work and in turn was compensated according to the result
of his efforts. By the nature of the business of soliciting insurance, agents are normally
left free to devise ways and means of persuading people to take out insurance. There is
no prohibition, as contended by petitioner, for private respondent to work for as long as
he does not violate the Insurance Code. Although petitioner could have, theoretically,
disapproved any of private respondent’s transactions, what could be disapproved was
only the result of the work, and not the means by which it was accomplished.

The “control” which the above factors indicate did not sum up to the power to control
private respondent’s conduct in and mode of soliciting insurance. On the contrary, they
clearly indicate that the juridical element of control had been absent in this situation.
Thus, the Court is constrained to rule that no employment relationship had ever existed
between the parties.

Consulta v. Court of Appeals, G.R. No. 145443. March 18, 2005


Same; Same; Same; Respondent Pamana did not prohibit petitioner Consulta from engaging in any other
business, or from being connected with any other company, for as long as the business or company did
not compete with Pamana’s business.—Consulta’s appointment had an exclusivity provision. The
appointment provided that Consulta must represent Pamana on an exclusive basis. She must not engage
directly or indirectly in activities of other companies that compete with the business of Pamana. However,
the fact that the appointment required Consulta to solicit business exclusively for Pamana did not mean
that Pamana exercised control over the means and methods of Consulta’s work as the term control is
understood in labor jurisprudence. Neither did it make Consulta an employee of Pamana. Pamana did not
prohibit Consulta from engaging in any other business, or from being connected with any other company,
for as long as the business or company did not compete with Pamana’s business.
Facts:

Pamana Philippines, Inc. ("Pamana") is engaged in health care business. Raquel P.


Consulta ("Consulta") was a Managing Associate of Pamana. Consulta's appointment
dated 1 December 1987 states:

We are pleased to formally confirm your appointment and confer upon you the authority as
MANAGING ASSOCIATE (MA) effective on December 1, 1987 up to January 2, 1988. Your
area of operation shall be within Metro Manila.

In this capacity, your principal responsibility is to organize, develop, manage, and maintain a
sales division and a full complement of agencies and Health Consultants (HealthCons) and
to submit such number of enrollments and revenue attainments as may be required of
your... position in accordance with pertinent Company policies and guidelines. In pursuit of
this objective, you are hereby tasked with the responsibilities of recruiting, training and
directing your Supervising Associates (SAs) and the Health Consultants under their
respective... agencies, for the purpose of promoting our corporate Love Mission.

In the performance of such duties, you are expected to uphold and promote the Company's
interests and good image and to abide by its principles and established norms of conduct
necessary and appropriate in the discharge of your functions. The authority as MA likewise
vests... upon you command responsibility for the actions of your SAs and HealthCons; the
Company therefore reserves the right to debit your account for any accountabilities/financial
obligations arising therefrom.

By your acceptance of this appointment, it is understood that you must represent the
Company on an exclusive basis, and must not engage directly or indirectly in activities, nor
become affiliated in official or unofficial capacity with companies or organizations which...
compete or have the same business as Pamana. It is further understood that his [sic]
self-inhibition shall be effective for a period of one year from date of official termination with
the Company arising from any cause whatsoever.

In consideration of your undertaking the assignment and the accompanying duties and
responsibilities, you shall be entitled to compensation computed as follows:
On Initial Membership Fee

Entrance Fee

5%

Medical Fee

6%

On Subsequent Membership Fee

6%

You are likewise entitled to participate in sales contests and such other incentives that may
be implemented by the Company.

This appointment is on a non-employer-employee relationship basis, and shall be in


accordance with the Company Guidelines on Appointment, Reclassification and Transfer of
Sales Associates.[3]

Sometime in 1987, Consulta negotiated with the Federation of Filipino Civilian Employees
Association ("FFCEA") working at the United States Subic Naval Base for a Health
Care Plan for the FFCEA members. Pamana issued Consulta a Certification[4] dated 23
November 1987, as follows:

This certifies that the Emerald Group under Ms. Raquel P. Consulta, as Managing
Consultant, is duly authorized to negotiate for and in behalf of PAMANA with the Federation
of Filipino Civilian Employees Association covering all U.S. facilities in the Philippines, the...
coverage of FFCEA members under the Pamana Golden Care Health Plans.

Upon such negotiation and eventual execution of the contract agreements, entitlements of
all benefits due the Emerald Group in it's [sic] entirely including it's [sic] Supervising
Consultants and Health Consultants, by of commissions, over-rides and other package of
benefits is... hereby affirmed, obligated and confirmed as long as the contracts negotiated
and executed are in full force and effect, including any and all renewals made. And
provided further that the herein authorized consultants remain in active status with the
Pamana Golden Care... sales group.[5]

On 4 March 1988, Pamana and the U.S. Naval Supply Depot signed the FFCEA account.
Consulta, claiming that Pamana did not pay her commission for the FFCEA account, filed a
complaint for unpaid wages or commission against Pamana, its President Razul Z.
Requesto ("Requesto"),... and its Executive Vice-President Aleta Tolentino

In a Decision promulgated on 23 June 1993, Labor Arbiter Alex Arcadio Lopez ruled, as
follows:

ACCORDINGLY, respondent is hereby ordered to pay complainant her unpaid commission


to be computed as against actual transactions between respondent PAMANA and the
contracting Department of U.S. Naval Supply Depot upon presentation of pertinent
document.

In its Decision promulgated on 28 April 2000, the appellate court reversed the NLRC
Decision. The appellate court ruled that Consulta was a commission agent, not an
employee of Pamana. The appellate court also ruled that Consulta should have litigated
her claim for... unpaid commission in an ordinary civil action.

Issues:

The issues are:


Whether Consulta was an employee of Pamana.

Whether the Labor Arbiter had jurisdiction over Consulta's claim for unpaid commission.

The Ruling of the Court

Ruling:

We affirm the Decision of the appellate court. Consulta was an independent agent and not
an employee of Pamana.

Principles:

In the present case, the power to control is missing. Pamana tasked Consulta to organize,
develop, manage, and maintain a sales division, submit a number of enrollments and
revenue attainments in accordance with company policies and guidelines, and to recruit,
train and... direct her Supervising Associates and Health Consultants.[12] However, the
manner in which Consulta was to pursue these activities was not subject to the control of
Pamana. Consulta failed to show that she had to report for work at definite... hours. The
amount of time she devoted to soliciting clients was left entirely to her discretion. The
means and methods of recruiting and training her sales associates, as well as the
development, management and maintenance of her sales division, were left to her... sound
judgment.

Clearly, the Managing Associates only received suggestions from Pamana on how to go
about their recruitment and sales activities. They could adopt the suggestions but the
suggestions were not binding on them. They could adopt other methods that they deemed
more... effective.

Further, the Managing Associates had to ask the Management of Pamana to shoulder half
of the advertisement cost for their recruitment campaign. They shelled out their own
resources to bolster their recruitment. They shared in the payment of the salaries of their...
secretaries. They gave cash incentives to their sales associates from their own pocket.
These circumstances show that the Managing Associates were independent contractors,
not employees, of Pamana.

Finally, Pamana paid Consulta not for labor she performed but only for the results of her
labor.[16] Without results, Consulta's labor was her own burden and loss. Her right to
compensation, or to commission, depended on the tangible results of... her work[17] -
whether she brought in paying recruits. Consulta's appointment paper provides:

In consideration of your undertaking the assignment and the accompanying duties and
responsibilities, you shall be entitled to compensation computed as follows:

On Initial Membership Fee

Entrance Fee

5%

Medical Fee

6%

On Subsequent Membership Fee

6%
You are likewise entitled to participation in sales contests and such other incentives that
may be implemented by the Company.[18]The Guidelines on Appointment of Associates
show that a Managing Associate received the following commissions and... bonuses:

3. Compensation Package of Regular MAs

Regular MAs shall be entitled to the following compensation and benefits:

3.1 Compensation... a) Personal Production

Individual/Family

Institutional Acct.

commission

30%

30%... bonus

40%

-... b) Group Production

... overriding commission

6%

6%... bonus

5%

3.2 Benefits

Participation in all sales contests corresponding to the MA position plus any such other
benefits as may be provided for the MA on regular status

Aside from commissions, bonuses and other benefits that depended solely on actual sales,
Pamana did not pay Consulta any compensation for managing her sales division, or for
recruiting and training her sales consultants. As a Managing Associate, she was only
entitled to... commissions, bonuses and other benefits, which depended solely on her sales
and on the sales of her group.

Consulta's appointment had an exclusivity provision. The appointment provided that


Consulta must represent Pamana on an exclusive basis. She must not engage directly or
indirectly in activities of other companies that compete with the business of Pamana.
However, the... fact that the appointment required Consulta to solicit business exclusively
for Pamana did not mean that Pamana exercised control over the means and methods of
Consulta's work as the term control is understood in labor jurisprudence.[20] Neither did it
make

Consulta an employee of Pamana. Pamana did not prohibit Consulta from engaging in any
other business, or from being connected with any other company, for as long as the
business or company did not compete with Pamana's business.

The prohibition applied for one year after the termination of the contract with Pamana. In
one of their meetings, one of the Managing Associates reported that he was transferring his
sales force and account from another company to Pamana.[21] The... exclusivity provision
was a reasonable restriction designed to prevent similar acts prejudicial to Pamana's
business interest. Article 1306 of the Civil Code provides that "[t]he contracting parties may
establish such stipulations, clauses, terms and conditions as they may... deem convenient,
provided they are not contrary to law, morals, good customs, public order, or public policy."
Comparing the Labor Code concept of "control" with the "control" that must necessarily
exist in a principal-agent relationship

Tongko v. The Manufacturers Life Insurance Company, De Dios, G.R. No. 167622, June 29,
2010
The main issue of whether an agency or an employment relationship exists depends on the incidents
of the relationship. The Labor Code concept of "control" has to be compared and distinguished with
the "control" that must necessarily exist in a principal-agent relationship. The principal cannot but
also have his or her say in directing the course of the principal-agent relationship, especially in cases
where the company-representative relationship in the insurance industry is an agency.

Tongko v. The Manufacturers Life Insurance Company, De Dios, G.R. No. 167622, January
25, 2011
Labor Law; Agency; Employer-Employee Relationship; Control over the performance of the task of
one providing service—both with respect to the means and manner, and the results of the service—is the
primary element in determining whether an employment relationship exists; Manulife’s control fell short
of this norm and carried only the characteristic of the relationship between an insurance company and its
agents, as defined by the Insurance Code and by the law of agency under the Civil Code.—Control over
the performance of the task of one providing service—both with respect to the means and manner, and the
results of the service—is the primary element in determining whether an employment relationship exists.
We resolve the petitioner’s Motion against his favor since he failed to show that the control Manulife
exercised over him was the control required to exist in an employer-employee relationship; Manulife’s
control fell short of this norm and carried only the characteristic of the relationship between an insurance
company and its agents, as defined by the Insurance Code and by the law of agency under the Civil Code.
FACTS: Taking from the November 2008 decision, the facts are as
follows:

Manufacturers Life Insurance, Co. is a domestic corporation engaged in


life insurance business. De Dios was its President and Chief Executive
Officer. Petitioner Tongko started his relationship with Manulife in 1977
by virtue of a Career Agent's Agreement.
Pertinent provisions of the agreement state that:

It is understood and agreed that the Agent is an independent contractor


and nothing contained herein shall be construed or interpreted as
creating an employer-employee relationship between the Company and
the Agent.

a) The Agent shall canvass for applications for Life Insurance, Annuities,
Group policies and other products offered by the Company, and collect,
in exchange for provisional receipts issued by the Agent, money due or to
become due to the Company in respect of applications or policies
obtained by or through the Agent or from policyholders allotted by the
Company to the Agent for servicing, subject to subsequent confirmation
of receipt of payment by the Company as evidenced by an Official
Receipt issued by the Company directly to the policyholder.

b) The Company may terminate this Agreement for any breach or


violation of any of the provisions hereof by the Agent by giving written
notice to the Agent within fifteen (15) days from the time of the discovery
of the breach. No waiver, extinguishment, abandonment, withdrawal or
cancellation of the right to terminate this Agreement by the Company
shall be construed for any previous failure to exercise its right under any
provision of this Agreement.

c) Either of the parties hereto may likewise terminate his Agreement at


any time without cause, by giving to the other party fifteen (15) days
notice in writing.

Sometime in 2001, De Dios addressed a letter to Tongko, then one of the


Metro North Managers, regarding meetings wherein De Dios found
Tongko's views and comments to be unaligned with the directions the
company was taking. De Dios also expressed his concern regarding the
Metro North Managers' interpretation of the company's goals. He
maintains that Tongko's allegations are unfounded. Some allegations
state that some Managers are unhappy with their earnings, that they're
earning less than what they deserve and that these are the reasons why
Tonko's division is unable to meet agency development objectives.
However, not a single Manager came forth to confirm these allegations.
Finally, De Dios related his worries about Tongko's inability to push for
company development and growth.

De Dios subsequently sent Tongko a letter of termination in accordance


with Tongko's Agents Contract. Tongko filed a complaint with the NLRC
against Manulife for illegal dismissal, alleging that he had an
employer-employee relationship with De Dios instead of a revocable
agency by pointing out that the latter exercised control over him through
directives regarding how to manage his area of responsibility and setting
objectives for him relating to the business. Tongko also claimed that his
dismissal was without basis and he was not afforded due process. The
NLRC ruled that there was an employer-employee relationship as
evidenced by De Dios's letter which contained the manner and means by
which Tongko should do his work. The NLRC ruled in favor of Tongko,
affirming the existence of the employer-employee relationship.

The Court of Appeals, however, set aside the NLRC's ruling. It applied
the four-fold test for determining control and found the elements in this
case to be lacking, basing its decision on the same facts used by the
NLRC. It found that Manulife did not exert control over Tongko, there
was no employer-employee relationship and thus the NLRC did not have
jurisdiction over the case.

The Supreme Court reversed the ruling of the Court of Appeals and ruled
in favor of Tongko. However, the Supreme Court issued another
Resolution dated June 29, 2010, reversing its decision. Tongko filed a
motion for reconsideration, which is now the subject of the instant case.

ISSUE: Did the Supreme Court err in issuing the June 29, 2010
resolution, reversing its earlier decision that an
employer-employee relationship existed?

HELD: The Supreme Court finds no reason to reverse the June 29, 2010
decision. Control over the performance of the task of one providing
service both with respect to the means and manner, and the results of the
service is the primary element in determining whether an employment
relationship exists. The Supreme Court ruled petitioners Motion against
his favor since he failed to show that the control Manulife exercised over
him was the control required to exist in an employer-employee
relationship; Manulifes control fell short of this norm and carried only
the characteristic of the relationship between an insurance company and
its agents, as defined by the Insurance Code and by the law of agency
under the Civil Code.

In the Supreme Courts June 29, 2010 Resolution, they noted that there
are built-in elements of control specific to an insurance agency, which do
not amount to the elements of control that characterize an employment
relationship governed by the Labor Code.The Insurance Code provides
definite parameters in the way an agent negotiates for the sale of the
companys insurance products, his collection activities and his delivery of
the insurance contract or policy. They do not reach the level of control
into the means and manner of doing an assigned task that invariably
characterizes an employment relationship as defined by labor law.

To reiterate, guidelines indicative of labor law "control" do not merely


relate to the mutually desirable result intended by the contractual
relationship; they must have the nature of dictating the means and
methods to be employed in attaining the result. Tested by this norm,
Manulifes instructions regarding the objectives and sales targets, in
connection with the training and engagement of other agents, are among
the directives that the principal may impose on the agent to achieve the
assigned tasks.They are targeted results that Manulife wishes to attain
through its agents. Manulifes codes of conduct, likewise, do not
necessarily intrude into the insurance agents means and manner of
conducting their sales. Codes of conduct are norms or standards of
behavior rather than employer directives into how specific tasks are to be
done.

In sum, the Supreme Court found absolutely no evidence of labor law


control. DENIED.
Relationship transformed to employer-employer relations

Great Pacific Life Assurance Corporation v. NLRC, Ruiz, G.R. No. 80750-51 July 23, 1990
Labor Law; Employer-employee relationship; When employment deemed to be regular.—Article 280
of the Labor Code provides that “[t]he provisions of written agreement to the contrary notwithstanding
and regardless of the oral agreements of the parties, an employment shall be deemed to be regular where
the employee has been engaged to perform activities which are usually necessary or desirable in the usual
business or trade of the employer.
Same; Same; Same; Control test being the most crucial and determinative indicator of an
employer-employee relationship.—Furthermore, in determining who is considered an “employee”, the
Court has time and again applied the “four-fold” test, with control being the most crucial and
determinative indicator of an employer-employee relationship. The “employer” must have control (or
must have reserved the right to control) not only over the result of the “employee’s” work but also the
means and methods by which it is to be accomplished.
Same; Same; Same; Same; As correctly hold by public respondent, the relationships of the Ruiz
brothers and Grepalife were those of employer-employee.—Applying the above, the Court finds that, as
correctly held by public respondent, the relationships of the Ruiz brothers and Grepalife were those of
employer-employee.

0, respondent should be ordered to pay complainants Rodrigo Ruiz and Ernesto Ruiz separation
pay, equivalent to one-half month's salary for every year of service. [NLRC Resolution, p. 5; Rollo, p.
9; Emphasis supplied.]

Hence, the present petition.

The Solicitor General filed its comment on behalf of public respondent. Petitioner filed a reply
thereto. Subsequently, the Court resolved to give the petition due course and to require the parties to
submit their memoranda. Petitioner and public respondent complied and duly submitted their
respective memoranda. On the other hand, private respondents did not file their comment and
memorandum.

The existence of valid grounds for private respondents' dismissal is not disputed herein, and
therefore the finding that the Ruiz brothers were dismissed for just cause is final. The only issues in
this petition are (1) Whether or not there was grave abuse of discretion on the part of public
respondent in holding that Ernesto and Rodrigo are employees of Grepalife; and (2) Whether or not
there was grave abuse of discretion on the part of public respondent in ordering the award of
separation pay to private respondents as sanction for Grepalife's failure to accord them due process
even though there was finding of just cause for their dismissal.

With respect to the first issue, the Court finds no grave abuse of discretion.

Grepalife contends that Rodrigo and Ernesto are agents, not employees, of the company by alleging
that they were hired under agency agreements, that they were not among the company's "organic
personnel" who handled technical and administrative functions of the company, that they were paid
on the basis of production/output (by way of commissions and bonuses, and not salaries), and that
they were neither under any form of control whatsoever as to hours of work nor were they "on call"
by the company. On the basis of the foregoing, Grepalife concluded that the relationship was one of
principal-agent and therefore, necessarily, it is the Civil Code and the Insurance Code which properly
govern the relationship, to the exclusion of the Labor Code.

This contention is devoid of merit.

Article 280 of the Labor Code provides that "[the provisions of written agreement to the contrary
notwithstanding and regardless of the oral agreements of the parties, an employment shall be
deemed to be regular where the employee has been engaged to perform activities which are usually
necessary or desirable in the usual business or trade of the employer. ..." Furthermore, in
determining who is considered an "employee', the Court has time and again applied the "four-fold"
test,* with control being the most crucial and determinative indicator of an employer-employee
relationship. The 'employer" must have control (or must have reserved the right to control) not only
over the result of the "employee's" work but also the means and methods by which it is to be
accomplished [Investment Planning Corp. of the Philippines v. SSS, G.R. No. L-19124, November
18, 1967, 21 SCRA 924; Mafinco Trading Corp. v. Ople, G.R. No. L-37790, March 25, 1976, 70
SCRA 139; Rosario Brothers, Inc. v. Ople, G.R. No. 53590, July 31, 1984, 131 SCRA 72;
Brotherhood Labor Unity Movement of the Philippines v. Zamora, G.R. No.

L-48645, January 7, 1987,147 SCRA 49; Grepalife v. NLRC, G.R. No. 73887, December 21, 1989].

Applying the above, the Court finds that, as correctly held by public respondent, the relationships of
the Ruiz brothers and Grepalife were those of employer-employee.

First, their work at the time of their dismissal as zone supervisor and district manager are necessary
and desirable to the usual business of the insurance company. They were entrusted with
supervisory, sales and other functions to guard Grepalife's business interests and to bring in more
clients to the company, and even with administrative functions to ensure that all collections, reports
and data are faithfully brought to the company.

Furthermore, it cannot be gainsaid that Grepalife had control over private respondents' performance
as well as the result of their efforts. A cursory reading of their respective functions as enumerated in
their contracts reveals that the company practically dictates the manner by which their jobs are to be
carried out. For instance, the District Manager must properly account, record and document the
company's funds spot-check and audit the work of the zone supervisors, conserve the company's
business in the district through 'reinstatements', follow up the submission of weekly remittance
reports of the debit agents and zone supervisors, preserve company property in good condition, train
understudies for the position of district manager, and maintain his quota of sales (the failure of which
is a ground for termination). On the other hand, a zone supervisor must direct and supervise the
sales activities of the debit agents under him, conserve company property through "reinstatements",
undertake and discharge the functions of absentee debit agents, spot-check the records of debit
agents, and insure proper documentation of sales and collections by the debit agents.

True, it cannot be denied that based on the definition of an "insurance agent" in the Insurance Code
[Art. 300] some of the functions performed by private respondents were those of insurance agents.
Nevertheless, it does not follow that they are not employees of Grepalife. The Insurance Code may
govern the licensing requirements and other particular duties of insurance agents, but it does not bar
the application of the Labor Code with regard to labor standards and labor relations.

Moreover, it is well-settled that the existence of an employer-employee relationship is ultimately a


question of fact, and such findings of fact of the labor arbiter and the NLRC shall be accorded not
only respect but even finality when supported by substantial evidence [RJL Martinez Fishing
Corporation v. NLRC, G.R. Nos. 63550-51, January 31, 1984, 127 SCRA 454; Asim v. Castro, G.R.
Nos. 75063-64, June 30, 1988, 163 SCRA 344; Murillo v. Sun Valley Realty, Inc., G.R. No. 67272,
June 30, 1988, 163 SCRA 271], as in this case.

With respect to the second issue, petitioner argues that private respondents are not entitled to
separation pay since there was clear finding of just cause for dismissal, and furthermore "neither the
law nor the rules implementing the same authorizes the award of separation pay as 'penalty."
[Petition, p. 8; Rollo, p. 25.]

Again, the contention is devoid of merit.

It must be emphasized that the monetary award fixed by public respondent, although erroneously
termed as "separation pay', was in fact a sanction for the employer's failure to observe the
procedural requirements of due process provided under Rule XIV, Secs. 2, 5 and 6 of the rules
implementing Batas Pambansa Blg. 130, and the parties' own covenant [Annex "A", Rollo, p. 38.]
The imposition of such a sanction is in consonance with the ruling in the case of Wenphil v. NLRC,
[G.R. No. 80587, February 8,1989,170 SCRA 69). The Court held therein that an indemnity, not
"separation pay", must be imposed on the employer for failure to observe the procedural
requirements of notice and hearing prior to the dismissal of an employee for just cause. Considering
the circumstances of the case at bar, petitioner must indemnify private respondents in the amount of
One Thousand Pesos (P1,000.00) each [See also Shoemart, Inc. v. NLRC, G.R. No. 74229, August
11, 1989].

IN VIEW OF THE FOREGOING, the decision of the NLRC is hereby MODIFIED insofar as the
award of "separation pay" is concerned. In lieu of "separation pay" petitioner Grepalife is hereby
ordered to indemnify private respondents Rodrigo Ruiz and Ernesto Ruiz the amount of One
Thousand Pesos (P1,000.00) each.
Insular Life Assurance Co., Ltd. v. NLRC, De Los Reyes, G.R. No. 119930 March 12, 1998
Labor Law; Employer-Employee Relationship; Records of the case are replete with telltale indicators of
an existing employeremployee relationship between the two parties despite written contractual
disavowals.—We reject the submissions of petitioner and hold that respondent NLRC acted appropriately
within the bounds of the law. The records of the case are replete with telltale indicators of an existing
employer-employee relationship between the two parties despite written contractual disavowals.
Same; Same; National Labor Relations Commission was correct in finding that private respondent
was an employee of petitioner, but this holds true only insofar as the management contract is
concerned.—Parenthetically, both petitioner and respondent NLRC treated the agency contract and the
management contract entered into between petitioner and De los Reyes as contracts of agency. We
however hold otherwise. Unquestionably there exist major distinctions between the two agreements.
While the first has the earmarks of an agency contract, the second is far removed from the concept of
agency in that provided therein are conditionalities that indicate an employer-employee relationship. The
NLRC therefore was correct in finding that private respondent was an employee of petitioner, but this
holds true only insofar as the management contract is concerned. In view thereof, the Labor Arbiter has
jurisdiction over the case.
Same; Same; It is axiomatic that the existence of an employeremployee relationship cannot be
negated by expressly repudiating it in the management contract and providing therein that the
“employee” is an independent contractor when the terms of the agreement clearly show otherwise.—It is
axiomatic that the existence of an employeremployee relationship cannot be negated by expressly
repudiating it in the management contract and providing therein that the “employee” is an independent
contractor when the terms of the agreement clearly show otherwise. For, the employment status of a
person is defined and prescribed by law and not by what the parties say it should be. In determining the
status of the management contract, the “four-fold test” on employment earlier mentioned has to be
applied.
Facts: On August 21, 1992 petitioner entered into an agency contract with respondent Pantaleon
Delos Reyes authorizing the latter to solicit within the Philippines applications for life insurance and
annuities for which he would be paid compensation in the form of commitment. The contract was
prepared by petitioner in its entirety and Delos Reyes merely signed his confirmity thereto. It
contained the stipulation that no employer-employee relationship shall be created between the
parties and that the agent shall be free to exercise his own judgement as to time, place and means of
soliciting insurance. Delos Reyes however was prohibited by petitioner from working for any other
life insurance company, and violation of this stipulation was sufficient ground for termination of the
contract. Aside from soliciting insurance for the petitioner, private respondent was required to
submit to the former all completed applications for insurance within 90 consecutive days, deliver
policies, receive and collect initial premiums and balances of first year premiums, renewal
premiums, deposits on applications and payments on policy loans. Private respondent was also
bound to turn over to the company immediately any and all sums of money collected by him. In a
written communication by petitioner to respondent Delos Reyes, the latter was urged to register
with the Social Security System (SSS) as a self-employed individual as provided under PD 1636. On
March 1, 1993, petitioner and private respondent entered into another contract where the latter was
appointed as acting, unit manager under its office — the Cebu DSO vs Private respondent
concurrently as agent and acting unit manager until he was notified by petitioner on November 18,
1993 that his services were terminated effective December 18, 1993. On November 7, 1994 he filed a
complaint before the labor arbiter on the ground that he was illegally dismissed and that he was not
paid his salaries and separation pay.

Issue: Whether or not there is an employer-employee relationship between the parties to entitle
jurisdiction of the case before the labor arbiter.

Held: Yes. It is axiomatic that existence of an employer-employee relationship cannot be negated by


expressly repudiating it in the management contract and providing therein that the employee is an
independent contractor when the terms of the agreement clearly shows otherwise. For, the
employment status of a person is defined and prescribed by law and not by what the parties say it
should be. In determining the status of the management contract, the “four-fold test” on
employment earlier mentioned has to be applied.
Unlike Basiao, herein respondent Delos Reyes was appointed acting unit manager, not agency
manager. There is no evidence that to implement his obligations under the management contract,
Delos Reyes had organized an office. Petitioner in fact has admitted that it provided Delos Reyes a
place and a table at its office where he reported for and worked whenever he was not out in the
field. Placed under petitioner’s Cebu District Service Office, the unit was given a name by petitioner
– Delos Reyes and Associates — and assigned code no. 11753 and recruitment no. 109398. Under the
managership contract, Delos Reyes was obliged to work exclusively for petitioner in life insurance
solicitation and was imposed premium production quotas. Of course, the acting unit manager could
not underwrite other lines of insurance because his permanent certificate of authority was for life
insurance only and for no other. He was proscribed from accepting a managerial or supervisory
position. In any other office including the government without the written consent of petitioner.
Delos Reyes could only be promoted to permanent unit manager if he met certain requirements and
his promotion was recommended by the petitioner’s district manager and regional manager and
approved by its division manager. As acting unit manager, Delos Reyes performed functions beyond
mere solicitation of insurance business for petitioner. As found by the NLRC, he exercised
administrative functions which were necessary and beneficial to the business of insular life.

Exclusivity of service, control of assignment and removal of agents under private respondent’s unit,
collection of premiums, furnishing company facilities and materials as well as capital described as
unit development fund are but hallmarks of the management system in which herein private
respondent worked. This obtaining, there is no escaping the conclusion that private respondent
Pantaleon Delos Reyes was an employee of herein petitioner.

6. Application of the Two-Tiered Test

Francisco v. NLRC, G.R. No. 170087 August 31, 2006


Application of the four-fold test and the two- tiered test
Present Philippine law recognizes a two-tiered test. The first tier of the test is the four-fold
test. The second tier is the economics of the relationship test. But the latter test is used if
and only if there is going to be harshness in the results because of the strict
application of the four-fold test.

Labor Law; Employment; Control Test; The better approach would therefore be to adopt a two-tiered
test.—The better approach would therefore be to adopt a two-tiered test involving: (1) the putative
employer’s power to control the employee with respect to the means and methods by which the work is to
be accomplished; and (2) the underlying economic realities of the activity or relationship. This two-tiered
test would provide us with a framework of analysis, which would take into consideration the totality of
circumstances surrounding the true nature of the relationship between the parties. This is especially
appropriate in this case where there is no written agreement or terms of reference to base the relationship
on; and due to the complexity of the relationship based on the various positions and responsibilities given
to the worker over the period of the latter’s employment.

Same; Same; Same; Economic Activity; The determination of the relationship between employer and
employee depends upon the circumstances of the whole economic activity.—The determination of the
relationship between employer and employee depends upon the circumstances of the whole economic
activity, such as: (1) the extent to which the services performed are an integral part of the employer’s
business; (2) the extent of the worker’s investment in equipment and facilities; (3) the nature and degree
of control exercised by the employer; (4) the worker’s opportunity for profit and loss; (5) the amount of
initiative, skill, judgment or foresight required for the success of the claimed independent enterprise; (6)
the permanency and duration of the relationship between the worker and the employer; and (7) the degree
of dependency of the worker upon the employer for his continued employment in that line of business.

FACTS:

1995, Petitioner was hired by Kasei Corporation during its incorporation stage.
She was designated as Accountant and Corporate Secretary and was assigned
to handle all the accounting needs of the company. She was also designated as
Liaison Officer to the City of Makati to secure business permits, construction
permits and other licenses for the initial operation of the company.

Although she was designated as Corporate Secretary, she was not entrusted
with the corporate documents; neither did she attend any board meeting nor
required to do so. She never prepared any legal document and never
represented the company as its Corporate Secretary. 1996, petitioner was
designated Acting Manager. Petitioner was assigned to handle recruitment of
all employees and perform management administration functions; represent
the company in all dealings with government agencies, especially with the
BIR, SSS and in the city government of Makati; and to administer all other
matters pertaining to the operation of Kasei Restaurant which is owned and
operated by Kasei Corporation.

January 2001, petitioner was replaced by a certain Liza R. Fuentes as


Manager. Kasei Corporation reduced her salary, she was not paid her mid-year
bonus allegedly because the company was not earning well. On October 2001,
petitioner did not receive her salary from the company. She made repeated
follow-ups with the company cashier but she was advised that the company
was not earning well. Eventually she was informed that she is no longer
connected with the company.

Since she was no longer paid her salary, petitioner did not report for work and
filed an action for constructive dismissal before the labor arbiter. Private
respondents averred that petitioner is not an employee of Kasei Corporation.
They alleged that petitioner was hired in 1995 as one of its technical
consultants on accounting matters and act concurrently as Corporate
Secretary. As technical consultant, petitioner performed her work at her own
discretion without control and supervision of Kasei Corporation. Petitioner
had no daily time record and she came to the office any time she wanted and
that her services were only temporary in nature and dependent on the needs of
the corporation.

The Labor Arbiter found that petitioner was illegally dismissed, NLRC
affirmed with modification the Decision of the Labor Arbiter. On appeal, CA
reversed the NLRC decision. CA denied petitioner’s MR, hence, the present
recourse.

ISSUES:

1. WON there was an employer-employee relationship between


petitioner and private respondent; and if in the affirmative,
2. Whether petitioner was illegally dismissed.

RULING:
1. Generally, courts have relied on the so-called right of control test
where the person for whom the services are performed reserves a
right to control not only the end to be achieved but also the means to
be used in reaching such end. In addition to the standard of
right-of-control, the existing economic conditions prevailing
between the parties, like the inclusion of the employee in the
payrolls, can help in determining the existence of an
employer-employee relationship.

There are instances when, aside from the employer’s power to control the
employee, economic realities of the employment relations help provide a
comprehensive analysis of the true classification of the individual, whether as
employee, independent contractor, corporate officer or some other capacity.

It is better, therefore, to adopt a two-tiered test involving: (1) the employer’s


power to control; and (2) the economic realities of the activity or relationship.

The control test means that there is an employer-employee relationship when


the person for whom the services are performed reserves the right to control
not only the end achieved but also the manner and means used to achieve that
end.

There has to be analysis of the totality of economic circumstances of the


worker. Thus, the determination of the relationship between employer and
employee depends upon the circumstances of the whole economic activity,
such as: (1) the extent to which the services performed are an integral part of
the employer’s business; (2) the extent of the worker’s investment in
equipment and facilities; (3) the nature and degree of control exercised by the
employer; (4) the worker’s opportunity for profit and loss; (5) the amount of
initiative, skill, judgment or foresight required for the success of the claimed
independent enterprise; (6) the permanency and duration of the relationship
between the worker and the employer; and (7) the degree of dependency of the
worker upon the employer for his continued employment in that line of
business. The proper standard of economic dependence is whether the worker
is dependent on the alleged employer for his continued employment in that
line of business

By applying the control test, it can be said that petitioner is an employee of


Kasei Corporation because she was under the direct control and supervision of
Seiji Kamura, the corporation’s Technical Consultant. She reported for work
regularly and served in various capacities as Accountant, Liaison Officer,
Technical Consultant, Acting Manager and Corporate Secretary, with
substantially the same job functions, that is, rendering accounting and tax
services to the company and performing functions necessary and desirable for
the proper operation of the corporation such as securing business permits and
other licenses over an indefinite period of engagement. Respondent
corporation had the power to control petitioner with the means and methods
by which the work is to be accomplished.

Under the economic reality test, the petitioner can also be said to be an
employee of respondent corporation because she had served the company for
6 yrs. before her dismissal, receiving check vouchers indicating her
salaries/wages, benefits, 13th month pay, bonuses and allowances, as well as
deductions and Social Security contributions from. When petitioner was
designated General Manager, respondent corporation made a report to the
SSS. Petitioner’s membership in the SSS evinces the existence of an
employer-employee relationship between petitioner and respondent
corporation. The coverage of Social Security Law is predicated on the
existence of an employer-employee relationship.

2. The corporation constructively dismissed petitioner when it reduced


her. This amounts to an illegal termination of employment, where
the petitioner is entitled to full backwages

A diminution of pay is prejudicial to the employee and amounts to


constructive dismissal. Constructive dismissal is an involuntary resignation
resulting in cessation of work resorted to when continued employment
becomes impossible, unreasonable or unlikely; when there is a demotion in
rank or a diminution in pay; or when a clear discrimination, insensibility or
disdain by an employer becomes unbearable to an employee. Petition is
GRANTED.

7. Economic dependency test

Sevilla v. Court of Appeals, G.R. Nos. L-41182-3, April 15, 1988


Labor; Employer-employee relation; No uniform test to determine the existence of an
employer-employee relation; Court relied on the so-called right of control test, the existing economic
conditions prevailing between the parties.—In this jurisdiction, there has been no uniform test to
determine the existence of an employer-employee relation. In general, we have relied on the so-called
right of control test, “where the person for whom the services are performed reserves a right to control not
only the end to be achieved but also the means to be used in reaching such end.” Subsequently, however,
we have considered, in addition to the standard or right-of-control, the existing economic conditions
prevailing between the parties, like the inclusion of the employee in the payrolls, in determining the
existence of an employer-employee relationship.
FACTS:
In 1960, the Tourist World Services Inc. (TWS) and Sevilla entered into a lease contract for the
use as branch office. In the said contract, both parties were held solidarily liable for the prompt
payment of the monthly rental agreed on. When the branch office was opened, it was run by
appellant Sevilla wherein any airline fare brought in on her efforts, 4% of that would go to her
and 3% was to be withheld by TWS.
The TWS appears to have been informed that Sevilla was connected with a rival firm, the
Philippine Travel Bureau, and, since the branch office was anyhow losing, the TWS considered
closing down its office. The premises were locked and neither the appellant Sevilla nor any of
her employees could enter, a complaint was filed by the herein appellants against the appellees
with a prayer for the issuance of mandatory preliminary injunction.
In the appeal, Lina Sevilla claims she was not an employee of the TWS to the end that her
relationship with TWS was one of a joint business venture. She declares that she did not
receive any salary from TWS and only earned commissions. Sevilla likewise claimed that she
shared in the expenses maintaining the office and TWS shouldered the rental in consideration
for the 3% split in the commissions procured.
TWS contend that the appellant was an employee of the appellee Tourist World Service, Inc.
and as such was designated manager and she had no say on the lease executed.

ISSUE:
Whether or not appellant Sevilla was in a joint venture with TWS or at least its agent coupled
with an interest which could not be terminated or revoked unilaterally by TWS.

HELD:
It is the Court’s considered opinion, that when the petitioner, Lina Sevilla, agreed to (wo)man the
private respondent, Tourist World Service, Inc.’s Ermita office, she must have done so pursuant
to a contract of agency. It is the essence of this contract that the agent renders services “in
representation or on behalf of another.” In the case at bar, Sevilla solicited airline fares, but she
did so for and on behalf of her principal, Tourist World Service, Inc. As compensation, she
received 4% of the proceeds in the concept of commissions. And as we said, Sevilla herself,
based on her letter of November 28, 1961, presumed her principal’s authority as owner of the
business undertaking. We are convinced, considering the circumstances and from the
respondent Court’s recital of facts, that the parties had contemplated a principal-agent
relationship, rather than a joint management or a partnership.
The agency that we hereby declare to be compatible with the intent of the parties, cannot be
revoked at will. The reason is that it is one coupled with an interest, the agency having been
created for the mutual interest of the agent and the principal. Accordingly, the revocation
complained of should entitle the petitioner, Lina Sevilla, to damages.
And apparently, Sevilla herself did not recognize the existence of such a relation. In her letter of
November 28, 1961, she expressly “concedes your [Tourist World Service, Inc.’s] right to stop
the operation of your branch office,” in effect, accepting Tourist World Service, Inc.’s control
over the manner in which the business was run. A joint venture, including a partnership,
presupposes generally a parity of standing between the joint co-venturers or partners, in which
each party has an equal proprietary interest in the capital or property contributed and where
each party exercises equal rights in the conduct of the business. Furthermore, the parties did
not hold themselves out as partners, and the building itself was embellished with the electric
sign “Tourist World Service, Inc.,” in lieu of a distinct partnership name.

DISPOSITIVE PORTION:
WHEREFORE, the Decision promulgated on January 23, 1975 as well as the Resolution issued
on July 31, 1975, by the respondent Court of Appeals is hereby REVERSED and SET ASIDE.
The private respondent, Tourist World Service, Inc., and Eliseo Canilao, are ORDERED jointly
and severally to indemnify the petitioner, Lina Sevilla, the sum of P25,000.00 as and for moral
damages, the sum of P10,000.00, as and for exemplary damages, and the sum of P5,000.00,
as and for nominal and/or temperate damages.

DOCTRINE:
The agency that we hereby declare to be compatible with the intent of the parties, cannot be
revoked at will. The reason is that it is one coupled with an interest, the agency having been
created for the mutual interest of the agent and the principal.

Orozco v. Court of Appeals, G. R. No. 155207, August 13, 2008, supra


Same; Same; Same; Same; Economic Reality Test; In our jurisdiction, the benchmark of economic reality
in analyzing possible employment relationships for purposes of applying the Labor Code ought to be the
economic dependence of the worker on his employer.—Aside from the control test, this Court has also
used the economic reality test. The economic realities prevailing within the activity or between the parties
are examined, taking into consideration the totality of circumstances surrounding the true nature of the
relationship between the parties. This is especially appropriate when, as in this case, there is no written
agreement or contract on which to base the relationship. In our jurisdiction, the benchmark of economic
reality in analyzing possible employment relationships for purposes of applying the Labor Code ought to
be the economic dependence of the worker on his employer.
8. Employer-employee relationship in job contracting v. labor-only contracting

Vigilia v. Philippine College of Criminology, Inc. G. R. No. 200094, June 10, 2013
Labor Law; Labor-Only Contracting; Solidary Liability; The basis of the solidary liability of the
principal with those engaged in labor-only contracting is the last paragraph of Article 106 of the Labor
Code, which in part provides: “In such cases [labor-only contracting], the person or intermediary shall
be considered merely as an agent of the employer who shall be responsible to the workers in the same
manner and extent as if the latter were directly employed by him.”—The NLRC and the CA correctly
ruled that the releases, waivers and quitclaims executed by petitioners in favor of MBMSI redounded to
the benefit of PCCr pursuant to Article 1217 of the New Civil Code. The reason is that MBMSI is
solidarily liable with the respondents for the valid claims of petitioners pursuant to Article 109 of the
Labor Code. As correctly pointed out by the respondents, the basis of the solidary liability of the principal
with those engaged in labor-only contracting is the last paragraph of Article 106 of the Labor Code,
which in part provides: “In such cases [labor-only contracting], the person or intermediary shall be
considered merely as an agent of the employer who shall be responsible to the workers in the same
manner and extent as if the latter were directly employed by him.”
Facts:

The petitioners work for the Philippine College of Criminology Inc. (PCCr) as janitors, janitress
and supervisor in its maintenance department. The petitioners were made to understand by the
respondent PCCr that they are under the Metropolitan Building Services, Inc. (MBMSI) which is
a corporation engaged in providing janitorial services. PCCr terminated the services of MBMSI
on 2009 which resulted in the dismissal of the petitioners. An illegal dismissal complaint was
then filed against PCCr by the petitioners contending that it is their real employer and not
MBMSI. Subsequently, the PCCr submitted to the Labor Arbiter waivers, releases and
quitclaims that were executed by the petitioners in favor to MBMSI.

The Labor Arbiter and NLRC ruled in favor of the petitioner, however upon filing the petition for
review on certiorari before the Court of Appeals, the CA ruled that the quitclaims, releases and
waivers executed by the petitioners in favor to MBMSI redounds to the benefit of PCCr by virtue
of solidary liability under Article 1217 of the NewCivil Code. The petitioners contend that under
Article 106 of the Labor Code a labor-only contractor's liability is not solidary as it is the
employer who should be directly responsible to the supplied worker.

Issue

Whether or not the quitclaims, releases and waivers executed by the petitioners in favor to
MBMSI redounds to the benefit of PCCr?

Held

Yes.

The Supreme Court held that the basis of the solidary liability of the principal with those
engaged in labor-only contracting is the last paragraph of Article 106 of the Labor Code that
provides, "In such cases of labor-only contracting, the person or intermediary shall be
considered merely as an agent of the employer who shall be responsible to the workers in the
same manner and extent as if the latter were directly employed by him."

It also pointed out D.O. No. 18-A, s. 2011 section 27 providing for the effects of labor-only
contracting "where upon the finding by competent authority of labor-only contracting shall render
the principal jointly and severally liable with the contractor to the latter's employees, in the same
manner and extent that the principal is liable to employees directly hired by him/her, as provided
in Article 106 of the Labor Code."

Hence, the PCCr's solidary liability was already expunged by virtue of the releases, waivers and
quitclaims executed by the petitioners in favor of MBMSI by virtue of Article 1217 of the Civil
Code providing that "payment made by one of the solidary debtors extinguishes the obligation."
9. Independent contracting under D.O. 174-17 (D.O. 18-A) v. independent contractor
recognized by jurisprudence

Fuji Television Network Inc. v. Espiritu, G. R. No. 204944-45, December 3, 2014, supra
Same; Same; Independent Contractors; There is no employer-employee relationship between the
contractor and principal who engages the contractor’s services, but there is an employer-employee
relationship between the contractor and workers hired to accomplish the work for the principal.—In
Department Order No. 18-A, Series of 2011, of the Department of Labor and Employment, a contractor is
defined as having: Section 3. . . . . . . . (c) . . . an arrangement whereby a principal agrees to put out or
farm out with a contractor the performance or completion of a specific job, work or service within a
definite or predetermined period, regardless of whether such job, work or service is to be performed or
completed within or outside the premises of the principal. This department order also states that there is a
trilateral relationship in legitimate job contracting and subcontracting arrangements among the principal,
contractor, and employees of the contractor. There is no employer-employee relationship between the
contractor and principal who engages the contractor’s services, but there is an employer-employee
relationship between the contractor and workers hired to accomplish the work for the principal.
Same; Same; Same; Since no employer-employee relationship exists between independent
contractors and their principals, their contracts are governed by the Civil Code provisions on contracts
and other applicable laws.—Since no employer-employee relationship exists between independent
contractors and their principals, their contracts are governed by the Civil Code provisions on contracts
and other applicable laws. A contract is defined as “a meeting of minds between two persons whereby one
binds himself, with respect to the other, to give something or to render some service.” Parties are free to
stipulate on terms and conditions in contracts as long as these “are not contrary to law, morals, good
customs, public order, or public policy.” This presupposes that the parties to a contract are on equal
footing. They can bargain on terms and conditions until they are able to reach an agreement.
Same; Same; Same; Employees under fixed-term contracts cannot be independent contractors
because in fixed-term contracts, an employer-employee relationship exists.—Fuji’s argument that Arlene
was an independent contractor under a fixed-term contract is contradictory. Employees under fixed-term
contracts cannot be independent contractors because in fixed-term contracts, an employer-employee
relationship exists. The test in this kind of contract is not the necessity and desirability of the employee’s
activities, “but the day certain agreed upon by the parties for the commencement and termination of the
employment relationship.” For regular employees, the necessity and desirability of their work in the usual
course of the employer’s business are the determining factors. On the other hand, independent contractors
do not have employer-employee relationships with their principals.
Same; Same; Same; Wages should not be the conclusive factor in determining whether one is an
employee or an independent contractor.—The Court of Appeals did not err when it relied on the ruling in
Dumpit-Murillo v. Court of Appeals, 524 SCRA 290 (2007), and affirmed the ruling of the National Labor
Relations Commission finding that Arlene was a regular employee. Arlene was hired by Fuji as a news
producer, but there was no showing that she was hired because of unique skills that would distinguish her
from ordinary employees. Neither was there any showing that she had a celebrity status. Her monthly
salary amounting to US$1,900.00 appears to be a substantial sum, especially if compared to her salary
when she was still connected with GMA. Indeed, wages may indicate whether one is an independent
contractor. Wages may also indicate that an employee is able to bargain with the employer for better pay.
However, wages should not be the conclusive factor in determining whether one is an employee or an
independent contractor.

10. Kind of relationship in boundary system arrangement


Mode of Compensation; Not Determinative Of Er-Ee Relationship
Piece-rate, boundary, and pakyaw are merely methods of pay computation and do not
prove whether the payee is an employee or not.

Boundary-Hulog System
Under the boundary-hulog scheme, a dual juridical relationship was created: that of
employer-employee and vendor-vendee. The boundary system is a scheme by an
owner/operator engaged in transporting passengers as a common carrier to primarily
govern the compensation of the driver, that is, the latter’s daily earnings are remitted to
the owner/operator less the excess of the boundary which represents the driver’s
compensation. Under this system, the owner/operator exercises control and supervision
over the driver. (Villamaria v. CA and Bustamante, G.R. No. 165881, April 19, 2006)
The boundary-hulog contract between the jeepney owner and the jeepney driver does not
negate the employer-employee relationship between them.

Villarama v. Court of Appeals, GR NO. 165881, Apr 19, 2006


Labor Law; Common Carriers; Boundary System; Words and Phrases; Jeepney
owner/operator-driver relationship under the boundary system is that of employer-employee and not
lessor-lessee; The boundary system is a scheme by an owner/operator engaged in transporting
passengers as a common carrier to primarily govern the compensation of the driver, that is, the latter’s
daily earnings are remitted to the owner/operator less the excess of the boundary which represents the
driver’s compensation.—As early as 1956, the Court ruled in National Labor Union v. Dinglasan, 98 Phil.
649 (1956), that the jeepney owner/operator-driver relationship under the boundary system is that of
employer-employee and not lessor-lessee. This doctrine was affirmed, under similar factual settings, in
Magboo v. Bernardo, 7 SCRA 952 (1963), and Lantaco, Sr. v. Llamas, 108 SCRA 502 (1981), and was
analogously applied to govern the relationships between auto-calesa owner/operator and driver, bus
owner/operator and conductor, and taxi owner/operator and driver. The boundary system is a scheme by
an owner/operator engaged in transporting passengers as a common carrier to primarily govern the
compensation of the driver, that is, the latter’s daily earnings are remitted to the owner/operator less the
excess of the boundary which represents the driver’s compensation. Under this system, the
owner/operator exercises control and supervision over the driver. It is unlike in lease of chattels where the
lessor loses complete control over the chattel leased but the lessee is still ultimately responsible for the
consequences of its use. The management of the business is still in the hands of the owner/operator, who,
being the holder of the certificate of public convenience, must see to it that the driver follows the route
prescribed by the franchising and regulatory authority, and the rules promulgated with regard to the
business operations. The fact that the driver does not receive fixed wages but only the excess of the
“boundary” given to the owner/operator is not sufficient to change the relationship between them.
Indubitably, the driver performs activities which are usually necessary or desirable in the usual business
or trade of the owner/operator.
Facts:
Oscar Villamaria, Jr. operated passenger jeepneys by employing drivers on a “boundary
basis.” In 1997, Villamaria agreed to sell the jeepney to driver Bustamante under the
“boundary-hulog scheme”. Their contract stipulated the prohibitions, compliance and
restrictions.
Bustamante continued driving the jeepney under the supervision and control of
Villamaria. But later he failed to comply with his obligations so that notice of compliance and
warning were ensued. Until in 2000, Villamaria took back the jeepney driven by Bustamante and
barred the latter from driving the vehicle. Hence, Bustamante filed a complaint for Illegal
Dismissal.
The LA ruled in his favor, but the NLRC reversed the Order for the reason that the juridical
relationship between Bustamante and Villamaria was that of vendor and vendee. However, the
CA affirmed the LA on the ground that the relationship between Villamaria and Bustamante was
dual: that of vendor-vendee and employer-employee.
Villamaria averred that their contract was a combination of vendor-vendee and
employer-employee because they had clearly entered into a conditional deed of sale over the
jeepney so that their employer-employee relationship had been transformed into that of
vendor-vendee.
Issue:
WoN the existence of a boundary-hulog agreement negates the employer-employee
relationship between the vendor and vendee.
Ruling:
No, The Kasunduan did not extinguish the employer-employee relationship of the parties
extant before the execution of said deed.
Under the boundary-hulog scheme incorporated in the Kasunduan, a dual juridical
relationship was created between petitioner and respondent: that of employer-employee and
vendor-vendee.
The fact that the driver does not receive fixed wages but only the excess of the
“boundary” given to the owner/operator is not sufficient to change the relationship between
them. Indubitably, the driver performs activities which are usually necessary or desirable in the
usual business or trade of the owner/operator.
Thus, the petition is denied.
11. Working scholar

Section 14, Rule X Book III, IRR

Filamer Christian Institute v. Court of Appeals, G.R. No. 75112 October 16, 1990
Torts; Quasi-Delict; Even assuming that an employer-employee relationship exists between Filamer
and Funtecha, still, Filamer cannot be made liable for the damages sustained by the victim, considering
that at the time of the accident, Funtecha was not acting within the scope of his employment.—But even if
we were to concede the status of an employee on Funtecha, still the primary responsibility for his
wrongdoing cannot be imputed to petitioner Filamer for the plain reason that at the time of the accident,
it has been satisfactorily shown that Funtecha was not acting within the scope of his supposed
employment. His duty was to sweep the school passages for two hours every morning before his regular
classes. Taking the wheels of the Pinoy jeep from the authorized driver at 6:30 in the evening and then
driving the vehicle in a reckless manner resulting in multiple injuries to a third person were certainly not
within the ambit of his assigned tasks. In other words, at the time of the injury, Funtecha was not engaged
in the execution of the janitorial services for which he was employed, but for some purpose of his own. It
is but fair therefore that Funtecha should bear the full brunt of his tortious negligence. Petitioner Filamer
cannot be made liable for the damages he had caused.

The legal issue in this appeal is whether or not the term "employer" as used in Article 2180 is
applicable to petitioner Filamer with reference to Funtecha.

7
In disclaiming liability, petitioner Filamer has invoked the provisions of the Labor Code, specifically
Section 14, Rule X of Book III which reads:

Sec. 14. Working scholars. — There is no employer-employee relationship between students on the
one hand, and schools, colleges or universities on the other, where students work for the latter in
exchange for the privilege to study free of charge; provided the students are given real opportunity,
including such facilities as may be reasonable, necessary to finish their chosen court under such
arrangement. (Emphasis supplied).

It is manifest that under the just-quoted provision of law, petitioner Filamer cannot be considered as
Funtecha's employer. Funtecha belongs to that special category of students who render service to
the school in exchange for free tuition Funtecha worked for petitioner for two hours daily for five days
a week. He was assigned to clean the school passageways from 4:00 a.m. to 6:00 a.m. with
sufficient time to prepare for his 7:30 a.m. classes. As admitted by Agustin Masa in open court,
Funtecha was not included in the company payroll.

Facts:

Private respondent Potenciano Kapunan, Sr., an eighty-two-year old retired


schoolteacher (now deceased), was struck by the Pinoy jeep owned by petitioner Filamer
and driven by its alleged employee, Funtecha, as Kapunan, Sr. was walking along Roxas
Avenue, Roxas City at 6:30 in the evening of October 20, 1977. As a result of the
accident, Kapunan, Sr. suffered multiple injuries for which he was hospitalized for a
total of twenty (20) days. At the time of the vehicular accident, only one headlight of the
jeep was functioning. Funtecha, who only had a student driver’s permit, was driving
after having persuaded Allan Masa, the authorized driver, to turn over the wheels to
him. The two fled from the scene after the incident. A tricycle driver brought the
unconscious victim to the hospital. The trial court rendered judgment finding not only
petitioner Filamer and Funtecha to be at fault but also Allan Masa, a non-party. Only
petitioner Filamer and third-party defendant Zenith Insurance Corporation appealed
the lower court’s judgment to the Court of Appeals and as a consequence, said lower
court’s decision became final as to Funtecha. For failure of the insurance firm to pay the
docket fees, its appeal was dismissed on September 18, 1984. On December 17, 1985, the
Appellate Court rendered the assailed judgment affirming the trial court’s decision in
toto. Hence the present recourse by petitioner Filamer.
Issue:

Whether or not the term “employer” as used in Article 2180 is applicable to


petitioner Filamer with reference to Funtecha.

Ruling:

The Court ruled that even if we were to concede the status of an employee on
Funtecha, still the primary responsibility for his wrongdoing cannot be imputed to
petitioner Filamer for the plain reason that at the time of the accident, it has been
satisfactorily shown that Funtecha was not acting within the scope of his supposed
employment. His duty was to sweep the school passages for two hours every morning
before his regular classes. Taking the wheels of the Pinoy jeep from the authorized
driver at 6:30 in the evening and then driving the vehicle in a reckless manner resulting
in multiple injuries to a third person were certainly not within the ambit of his assigned
tasks. At the time of the injury, Funtecha was not engaged in the execution of the
janitorial services for which he was employed, but for some purpose of his own. It is but
fair therefore that Funtecha should bear the full brunt of his tortious negligence.
Petitioner Filamer cannot be made liable for the damages he had caused. Furthermore,
the Court cited Section 14, Rule X of Book III of the Labor Code, under the Labor Code,
petitioner Filamer cannot be considered as Funtecha’s employer. Funtecha belongs to
that special category of students who render service to the school in exchange for free
tuition Funtecha worked for petitioner for two hours daily for five days a week. He was
assigned to clean the school passageways from 4:00 a.m. to 6:00 a.m. with sufficient
time to prepare for his 7:30 a.m. classes. As admitted by Agustin Masa in open court,
Funtecha was not included in the company payroll.

You might also like